hfhE∙ = ∙ = 6.62608 10 h J s

Transcrição

hfhE∙ = ∙ = 6.62608 10 h J s
Física Moderna – Capítulo 4 - Radiação de corpo negro e Efeito Fotoelétrico
Prof. Dr. Cláudio S. Sartori
Radiação. Lei de Stefan-Boltzmann. Corpo
Negro e Radiador Ideal.
Radiação
Quando se coloca uma das mãos em contato
direto com a superfície de um aquecedor d'água ou
radiador a vapor, o calor a atinge por condução através
das paredes do radiador. Se a mão for colocada acima
do mesmo, mas sem tocá-lo, o calor a atinge por meio
de correntes de convecção de ar aquecido, que se
movem para cima. Colocando-se em um dos lados do
radiador, ela ainda se torna quente, embora a condução
através do ar seja desprezível e esteja fora da trajetória
das correntes de convecção. A energia térmica, agora,
é transmitida por radiação.
O termo radiação refere-se á emissão contínua de
energia da superfície de todos os corpos. É chamada
energia radiante e tem a forma de ondas
eletromagnéticas. Essas ondas propagam-se com a
velocidade da luz e são transmitidas através do vácuo
ou do ar. (Na realidade, transmitem-se melhor no
vácuo, pois no ar são parcialmente absorvidas.)
Quando atingem um corpo que não lhes é transparente
como, por exemplo, a superfície da mão ou as paredes
de um quarto, são absorvidas.
23
E  h  f  h  c
Figura 3 – Variação do comprimento de
onda,
freqüência
e
energia
para
ondas
eletromagnéticas.
h é a constante de Planck:
h  6.62608 1034 J  s
Figura 4,5,6 – Ampliação da variação do
comprimento de onda, para ondas eletromagnéticas
a) e b) e Radiância espectral c):
a)
Ondas de Rádio:
b) Espectro visível.
c) Radiância Espectral indicando as curvas dos
resultados obtidos pelos os modelos de Planck (atual)
e Rayleigh-Jeans
Figura 1 – Ondas eletromagnéticas.
z
E
x
B
y
Figura 2 – Variação dos comprimentos de onda .
No início do século, Rayleigh, e também Jeans,
fizeram o cálculo da densidade de energia clássica da
23
Física Moderna – Capítulo 4 - Radiação de corpo negro e Efeito Fotoelétrico
Prof. Dr. Cláudio S. Sartori

radiação de cavidade. Podemos considerar uma
cavidade cúbica com paredes metálicas aquecidas
uniformemente à temperatura T contendo radiação
eletromagnética, formando ondas estacionárias nas
paredes da caixa. As paredes emitem radiação
eletromagnética na faixa térmica de freqüência.
Também denominamos esse modelo de corpo negro.
Esse cálculo mostrou uma séria divergência entre a
física clássica e os resultados experimentais.
 
   P  
n 0

 P  
n 0

 
n e
n 0

e


n
k T
n
k T
n 0
24
Como
k T
 n  n  h 
nh
n  h   k T
e

n 0 k  T

 

e

nh
k T
n 0
A fórmula de Rayleigh-Jeans para o corpo
negro é dada por:
8 kT
d
c3
: Freqüência da radiação
: densidade de energia
k: Constante de Boltzman: k  1,38  10 23
  k T
n 0

e
 n
 n
n 0
J
K
c: velocidade da luz: c  3,0  10
A função de distribuição de Boltzmann usada é
dada por:

 n   e
Note que:
8 m
s
d


ln   e n  
d  n  0


d
d
e
k T

e
e
 n  e n

0
Logo, podemos deduzir que::

 P   d
 n
n0

d
 

ln   e n  
d  n  0

   P   d
 n
n 0


k T
e o valor médio das energias do sistema:
 

2
 T ( )d 
P   
k T
h 
Chamando:  
k T
n 0

e
 n
n 0
   k T
0
Ao tentar solucionar essa discrepância entre a
teoria e a experiência, Planck foi levado a considerar a
hipótese de uma violação da lei da equipartição da
energia, sobre a qual a teoria clássica se baseava.
Planck supôs que a energia  poderia ter apenas
alguns valores discretos, em vez de qualquer valor, e
que os valores discretos fossem uniformemente
distribuídos, da forma:
  0,  , 2 ,3 , 4 ,

 
d
 

ln   e n  
d  n  0

   n  e n
n 0

e
 n
n 0
Assim:

  k T
 n   e
n 0

e
 n
 n
n 0
Assim:
24
Física Moderna – Capítulo 4 - Radiação de corpo negro e Efeito Fotoelétrico
Prof. Dr. Cláudio S. Sartori

  k T 
   n  e n
n 0

e
 n
n 0
  k  T   
  k  T 
h  d
 


ln   e n 
k  T d  n  0

  h  
25

e
 n
d
 

ln   e n 
d  n  0

d
 

ln   e n 
d  n  0

 1  e  e2  e3  
n 0

e
 n
 1  x  x 2  x3    x  e
n 0
1  x  x 2  x3   

e
 n
n 0

e
n 0
 n
1
1 x
Assim, Planck utilizou a fórmula que ele obteve
para a densidade de energia do espectro do corpo
negro, considerando modificações importantes na
distribuição clássica feita por Boltzmann; seu
resultado para a distribuição de energia foi dado por:
1

1  e
 1  e 
Figura 7 – (a) Modelo teórico da radiação de
corpo negro de Planck e dados experimentais.
1
 0   1  e 
  h 
1  e 

e
h
k T
1
Aqui h é a chamada constante de Planck e vale:
h  6,63  10 34 J  s .
Define-se também:  

e 
1  e 
e  e 
  h 
 
1  e  e 
1
  h  
e 1
h 
  h
e k T  1
h
E   


1
d
  h  
ln 1  e 
d
d
  h    1
ln 1  e 
d
  h 
h
2
z
E
x
B
y
A fórmula para a densidade de energia do
espectro do corpo negro, utilizando essa distribuição
de energia foi:
 T  d 
8 2
c3
h
e
h
k T
d
1
Esse é o espectro de corpo negro de Planck.
Se fizermos o espectro para comprimentos de
onda, teremos:

c
 d  

8hc
 T  d  5

1
e
hc
 k T
c
2
d
d
1
25
Física Moderna – Capítulo 4 - Radiação de corpo negro e Efeito Fotoelétrico
Prof. Dr. Cláudio S. Sartori
26
Figura 7 – Densidade de energia para diferentes toda a energia radiante emitida por um corpo tem
temperaturas, em função:
comprimentos de onda maiores do que esses. Tais
ondas são chamadas infravermelhas. Quando a
(b) Do comprimento de onda . Observe que temperatura aumenta, os comprimentos de onda
o pico do máximo desloca-se para a esquerda a medida desviam-se para valores menores. A 800°C, um
corpo emite bastante energia visível para ser
que a temperatura aumenta.
luminoso e aparece avermelhado. Ainda assim, a
maior parte da energia radiante ainda está no
infravermelho. A 3 000°C, que corresponde
aproximadamente à temperatura do filamento de uma
lâmpada incandescente, a energia radiante contém
uma proporção suficiente dos comprimentos de onda
mais curtos para parecer brancos.
Um pirômetro óptico é um dispositivo que
mede temperatura sem contacto com o corpo do qual
se pretende conhecer a temperatura. Geralmente este
termo é aplicado a instrumentos que medem
temperaturas superiores a 6000C. Uma utilização
típica é a medição da temperatura de metais
incandescentes em fundições.
Um dos pirómetros mais comuns é o de
absorção-emissão, que é utilizado para determinar a
temperatura de gases através da medição da radiação
emitida por uma fonte de referência, antes e depois
(c) Da freqüência . Observe o deslocamento da radiação incidir sobre o gás (que absorve parte da
do pico para a direita conforme o aumento da radiação). É através da análise das diferenças do
espectro do gás que se consegue determinar a sua
temperatura.
temperatura. Ambas as medições são feitas no

mesmo intervalo de comprimentos de onda.
Outra aplicação típica do pirómetro é a medição
-16
da temperatura de metais incandescentes. Olhando
3 10
pelo visor do pirômetro observa-se o metal,
-16
2.5 10
ajustando-se depois manualmente a corrente eléctrica
-16
2 10
que percorre um filamento que está no interior do
-16
1.5 10
pirómetro e aparece no visor. Quando a cor do
-16
filamento é idêntica à do metal, pode-se ler a
1 10
temperatura numa escala disposta junto ao elemento
-17
5 10
de ajuste da cor do filamento.

14
14
14
14
14
14A seguir indicamos o espectro solar obtido
1 10
2 10 3 10
4 10 5 10
6 10
experimentalmente, e veja a concordância com o
A energia radiante emitida por uma superfície, modelo da radiação de corpo negro de Planck.
por unidade de tempo e de área, depende da natureza e
Figura 8 - Radiância espectral solar e absorção
da temperatura do corpo. A baixas temperaturas, a taxa
de radiação é pequena e a energia radiante consiste atmosférica. Observe o modelo da radiância do
principalmente
em
comprimentos
de
onda corpo negro pontilhado em vermelho.
relativamente longos. À medida que a temperatura
aumenta, a taxa de radiação cresce rapidamente, sendo
diretamente proporcional à quarta potência da
temperatura absoluta. Por exemplo, um bloco de cobre
à temperatura de 100°C (373 K) irradia cerca de 0,03 J
• s -1 ou 0,03 W por cm2 de sua superfície, enquanto a
500°C (773 K), sua radiação é de 0,54 W por cm2.
Já a l 000°C (l 273 K), ela irradia cerca de 4
W por cm2. Essa taxa de radiação é cerca de 130 vezes
maior do que a uma temperatura de 100°C.
Em qualquer temperatura, a energia radiante
emitida é uma mistura de ondas de comprimento de
onda diferentes. Comprimentos de onda na faixa do
espectro visível variam de 0.4 . 10-6 m (violeta) até 0.7
.10 -6 m (vermelho). Na temperatura de 300°C, quase
26
Física Moderna – Capítulo 4 - Radiação de corpo negro e Efeito Fotoelétrico
Prof. Dr. Cláudio S. Sartori
x  4,96511423175275
x
hc
  k T
6.63 1034  3.0 108
 1,38 1023  T
4,96511423175275 
  T  2.898 103    m
Esta é conhecida como Lei do deslocamento
de Wien, onde o comprimento de onda da radiação,
, está em m.
Podemos também considerar o comprimento
de onda da radiação  em mm:
  T  2.898    mm 
27
 Lei de Stefan-Boltzmann
A experiência mostra que a taxa de radiação
da energia por uma superfície é proporcional à área
da superfície e à quarta potência da temperatura
absoluta T. Depende também da natureza da
superfície, descrita por um número adimensional e,
que está entre 0 e l. Assim, a relação pode ser
expressa por:

H   RT  d    T 4
 Lei do Deslocamento de Wien
Ao considerarmos a função de distribuição em
termos do comprimento de onda :
 T  d 
8hc

1
5
e
hc
 k T
d
1
Aqui h é a chamada constante de Planck e vale:
h  6,63  10 34 J  s
: Comprimento de onda da radiação
 : densidade de energia.
k: Constante de Boltzmann:
k  1,38  10 23 KJ
c: velocidade da luz:
0
H  A  e   T 4
onde ( é uma constante universal da Física,
chamada constante de Stefan-Boltzmann. Esta
relação foi deduzida por Josef Stefan (1835-1893)
com base nos resultados experimentais feitos por
John Tyndall (1820-1893) e, posteriormente,
derivada por considerações teóricas por Ludwig
Boltzmann (1844-1906).
A radiação de cavidade H é proporcional à
densidade de energia ·:
RT ( )  T ( )
c  3,0 10
8 m
s
Derivando em relação a :
T
  8 hc
1 

    5 hkcT

e
 1

Igualando a derivada a zero para encontrarmos
em qual comprimento de onda ocorrerá o máximo de
radiação, chega-se a:
hc
 hc

hc
  e  k T  1  e  k T 
0
5  k T


Chamando de: x 
hc
  k T
Figura 9 - Radiância espectral para
diversas temperaturas, mostrando o deslocamento
em , para a esquerda (indo para a região do UV), à
medida em que a temperatura aumenta.

Power , W m^2
1 10
8 10
6 10
4 10
2 10
14
6000. K
13
13
5000. K
13
13
4000. K
-7
-7
-7
-6
-6
-6
-6
2.5 10 5 10 7.5 10 1 10 1.25 101.5 101.75 10 2 10
Wavelength
-6
Geramos uma equação para f(x): f ( x)  e x  x  1  0
5
Aplicando métodos numéricos para a solução
desta equação, chega-se ao valor:
27
, m
Física Moderna – Capítulo 4 - Radiação de corpo negro e Efeito Fotoelétrico
Prof. Dr. Cláudio S. Sartori
absorvendo energia. Se qualquer objeto estiver mais
quente que o ambiente, sua taxa de emissão excederá
a de absorção. Haverá, assim, uma perda efetiva de
energia e o corpo se resfriará, a menos que seja
aquecido por um outro processo. Se, ao contrário, a
temperatura do corpo for mais baixa que a do
ambiente, sua taxa de absorção será maior que a de
emissão e a temperatura elevar-se-á. Quando o corpo
tiver a mesma temperatura que o ambiente, as duas
taxas tornar-se-ão iguais, não haverá perda ou ganho
de energia e a temperatura não variará.
28
Na equação anterior, H tem unidades de
potência (energia por unidade de tempo). Assim, no
SI, o tem unidades de W.m-2 K-4. O valor numérico de
Se um pequeno corpo de emissividade e
σ é:
estiver completamente envolvido por paredes cuja
temperatura é T, a taxa de absorção de energia
W
  5.6699 108 4 2
radiante, por unidade de área, pelo corpo será:
K m
H = Aea T4.
O número e, que caracteriza as propriedades
Daí, para um tal corpo a uma temperatura
de emissão de uma dada superfície, é chamado T1 e envolvido por paredes cuja temperatura é T2, a
emissividade. Em geral, ele é maior para superfícies taxa efetiva de perda (ou ganho) de energia, por
escuras e ásperas do que para superfícies lisas e claras. unidade de área, por radiação, é:
A emissividade de uma superfície polida de cobre é
H ef  A  e    T14  T24
aproximadamente de 0,3.
A emissão infravermelha de um corpo pode
Exemplo 3 - Uma fina placa quadrada de aço,
ser estudada por meio de uma câmara equipada com
com 10 cm de lado, é aquecida até a temperatura de
filme sensível ao infravermelho, ou com um aparelho
800°C. Sendo a emissividade igual a l, qual a taxa
semelhante, em princípio, a uma câmara de televisão
total de radiação de energia?
e sensível à radiação infravermelha. A fotografia
resultante é chamada termografïa.
Solução. A área total, incluindo ambos os lados
Uma vez que a emissão depende da
é 2 (0,1 m)2 = 0,02 m2. A temperatura que deve ser temperatura, a termografïa permite o estudo
colocada na anterior tem de ser a temperatura detalhado das distribuições de temperatura. Alguns
absoluta, isto é, 800°C = l 073 K. A equação dá, então, instrumentos atualmente são sensíveis a diferenças de
H = (0,02 m2) (l) (5,67 10-8 Wm-2K-4 ) (l 073 K)4
temperatura de até 0,1°C.
H = l 503 W.
A termografïa tem uma grande variedade de
Se a placa fosse aquecida por meio de um aplicações médicas importantes. Variações locais de
aquecedor elétrico, a potência de l 503 W teria que ser temperatura no corpo estão associadas a vários tipos
fornecida para manter a sua temperatura constante e de tumores, como câncer no seio e distúrbios no
igual a 800°C.
diâmetro de vasos, até um centímetro, podem ser
Se a superfície de qualquer corpo estiver detectados. Distúrbios vasculares que geram
continuamente emitindo energia radiante por que, anomalias locais de temperatura podem ser estudados
eventualmente, não irradia toda sua energia interna e e muitos outros exemplos poderiam ser citados.
resfria-se até a temperatura do zero absoluto (onde H =
0, pela equação). A resposta é que assim aconteceria
 Radiador Ideal
se, de certa maneira, não fosse fornecida energia ao
Imagine que as paredes de um recipiente
mesmo. No caso do filamento de uma lâmpada fechado sejam mantidas à temperatura T; e que vários
elétrica, a energia é fornecida eletricamente para corpos de diferentes emissividades sejam suspensos
compensar a energia radiada. Logo que se corta o sucessivamente
dentro
do
recipiente.
fornecimento de energia ao mesmo, ele se resfria Independentemente das temperaturas dos corpos que
rapidamente até atingir a temperatura ambiente. A são introduzidos, vê-se que, eventualmente, cada um
temperatura não desce mais porque o ambiente (as atinge a mesma temperatura Ti, isto é, os corpos
paredes e outros objetos no quarto) também está atingem o equilíbrio térmico com o ambiente.
radiando e certa quantidade desta energia radiante é Quando em equilíbrio térmico, o corpo emite energia
interceptada, absorvida e convertida em energia radiante. Parte desta energia é refletida e a restante,
interna. A mesma coisa é válida para todos os outros absorvida. Na ausência outro processo qualquer, a
objetos no quarto — estão simultaneamente emitindo e


28
Física Moderna – Capítulo 4 - Radiação de corpo negro e Efeito Fotoelétrico
Prof. Dr. Cláudio S. Sartori
29
energia absorvida elevará a temperatura do corpo
absorvente, mas como se observa que a temperatura
não varia, cada corpo deve emitir energia radiante na
mesma proporção que a absorve, Assim, um bom
absorvente é um bom emissor e um mal absorvente,
um mal emissor.
Mas como cada corpo deve absorver ou
refletir a energia radiante que o atinge, um mal
absorvente deve ser também um bom refletor. Assim,
um bom refletor é um mal emissor.
Esta é a razão das paredes das garrafas
térmicas serem prateadas. Tais recipientes são
fabricados com paredes duplas de vidro, entre as quais
se faz vácuo, de tal maneira que os fluxos de calor por
convecção condução são praticamente eliminados. A
fim de reduzir ao máximo as perdas por radiação,
cobrem-se a paredes com uma camada de prata, que é
altamente refletora e, portanto, muito má emissora.
Como um bom absorvente é um bom emissor,
o melhor emissor será aquele cuja superfície for mais
absorvente. Mas nenhuma superfície poderá absorver
maior quantidade de energia radiante do que a que
incide sobre ela. Qualquer superfície que absorve toda
a energia incidente será a melhor emissora possível
não refletiria energia radiante e apareceria, então, com
a cor negra (contanto que sua temperatura não seja tão
alta a torná-la autoluminosa) e, por isso, chama-se
superfície negra ideal; um corpo possuidor de tal
superfície é denominado corpo negro ideal, radiador
ideal ou simplesmente corpo negro.
Nenhuma superfície real é idealmente negra;
a mais aproximada é o negro-de-fumo, que reflete
apenas cerca de 1%, Entretanto, podem-se quase obter
as condições ideais de um corpo negro, fazendo-se
uma pequena abertura nas paredes de um recipiente
fechado. A energia radiante que entrar na abertura será
parcialmente absorvida pelas paredes interiores. Da
parte refletida, apenas uma quantidade muito pequena
escapa pela abertura, a restante sendo eventualmente
absorvida pelas paredes. Daí a abertura comportar-se
como um absorvente ideal.
Inversamente, a energia radiante emitida pelas
paredes ou por qualquer corpo dentro do recipiente
que escapa pela abertura, terá a mesma natureza que a
emitida por um radiador ideal, desde que as paredes
tenham uma temperatura uniforme. Esse fato é
importante quando se usa um pirômetro óptico. As
leituras de tal instrumento só serão corretas quando ele
for dirigido para um corpo negro. Se usado para medir
a temperatura de um lingote de ferro aquecido ao
rubro, ao ar livre, suas leituras seriam muito baixas,
pois o ferro é um emissor pior que o corpo negro. Se,
entretanto, o pirômetro for dirigido par o ferro
enquanto estiver ainda na fornalha, onde está
circundada por paredes à mesma temperatura, a
―condição de corpo negro‖ serão preenchidas e a
leitura será correia. A falha do ferro em emitir tão
efetiva mente quanto um corpo negro será justamente
compensada pela energia radiante que ele reflete.
A emissividade e de uma superfície
idealmente negra é igual à unidade. Para qualquer
superfície real à uma fração menor que l.
Exemplo 4 - Exemplo. Supondo que a
superfície total do corpo humano tenha l,2 m2 e que a
temperatura da superfície seja de 30°C = 303 K,
achar a taxa total de radiação de energia pelo corpo.
Solução. Surpreendentemente, para a radiação
infravermelha o corpo humano é uma ótima
aproximação
de um corpo negro ideal,
independentemente de pigmentação da pele. A taxa
da perda de energia é dada, usando e = l:
H = (1,2 m2) (l) (5,67 10-8 W • m-2 • K-4) (303
4
K) = 574 W.
Obviamente, esta perda é parcialmente
balanceada pela absorção de radiação, que depende
da temperatura do ambiente. A taxa líquida de
transferência de energia radiante é dada pela equação
anterior.
29
Física Moderna – Capítulo 4 - Radiação de corpo negro e Efeito Fotoelétrico
Prof. Dr. Cláudio S. Sartori
 Características
da
Eletromagnética e suas aplicações:
Faixa
(metros/hertz)
Variação
Específica
Ondas de Rádio
104 - 10-2 m/104 1010 Hz
c  f
ultra-low frequency
(ULF)
3 - 30 Hz
extremely low
frequency (ELF)
30 - 300 Hz
voice frequencies
(VF)
300 Hz - 3 kHz
c
f
c
f 


30
Radiação
very low frequency
(VLF)
ray 10-11 - 10-13 m/1019 1021 Hz
Gamma
3 - 30 kHz
Raios
gama
30 - 300 kHz
medium frequency 300 kHz - 3 MHz Submúltiplos
do
(MF)
comprimento
high frequency
3 - 30 MHz
de onda 
(HF)
Nanômetro
very high frequency 30 - 300 MHz
micrômetro
(VHF)
Angstron
ultra high
300 MHz - 3
frequency (UHF)
GHz
Infrared - Infravermelho
3 - 30 GHz
extremely high
frequency (EHF)
30 - 300 GHz
shortwave
see MF, HF
television
see VHF, UHF
microwave
30 cm - 1 mm/1300 GHz
10-3 - 10-6 m/1011 1014 Hz
 = 10
n = 10-9
-6
1000-30 m
far
30-3 m
middle
3-0.75 m
near
Ultraviolet –
Ultravioleta
X ray
Raio X
low frequency (LF)
super high frequency
(SHF)
Espectro visível
Visible
10-9 - 10-11 m/1017 1019 Hz
Notação
Valor
nm
µm
1 nm = 10-9m
1 µm = 10-6 m
1Å
1 Å = 10-10m
Espectro Eletromagnético de Radiação:
Região

0

cm
F
Hz
E
eV
A
Radio Rádio
> 109
> 10
< 3.109
< 10-5
MicrowaveMicroonda
109 106
10 - 0.01
3.109 –
3.1012
10-5 0.01
InfraredInfravermelho
106 7000
0.01 - 7
10-5
3.1012 4.3.1014
0.01 - 2
VisibleVisível
7000 4000
7.10-5 –
4.10-5
4.3.1014 7.5.1014
2-3
UltravioletUltravioleta
4000 10
4.10-5 10-7
7.5.1014
– 3.1017
3 - 103
X-Rays-Raio
X
10 0.1
10-7 - 10-
3.1017 –
3.1019
103 - 105
> 3.1019
> 105
5x10-7 m/2x1014 Hz
Red – Vermelho
770-622 nm
Orange – Laranja
622-597 nm
Yellow – Amarelo
597-577 nm
Green – Verde
577-492 nm
Blue – Azul
492-455 nm
Violet – Violeta
455-390 nm
10-7 - 10-8 m/1015 1016 Hz
UV-A
harmful)
(least
400-315 nm
UV-B
(more
harmful, absorbed
by ozone)
315-280 nm
UV-C
(most
harmful, but all
absorbed by air)
280-100 nm
near UV
light")
400-300 nm
("black
far UV
300-200 nm
vacuum UV
200-100 nm
Gamma
Rays-Raios
Gama
< 0.1
9
< 10-9
Ondas de rádio têm os comprimentos de
onda mais longos do espectro eletromagnético. Estas
ondas podem ser mais longas que um campo de
futebol ou tão pequeno quanto uma bola de futebol.
Ondas de rádio fazem mais do que trazer música para
seu rádio. Eles também levam sinais para sua
televisão e telefones celulares. As antenas fixadas em
sua televisão recebem o sinal, na forma de ondas
eletromagnéticas que são transmitidas da estação de
televisão. O sinal é exibido em sua tela de televisão.
Companhias de cabo têm antenas que
recebem ondas transmitidas de suas estações de
30
Física Moderna – Capítulo 4 - Radiação de corpo negro e Efeito Fotoelétrico
Prof. Dr. Cláudio S. Sartori
31
TELEVISÃO locais. O sinal é enviado então por um
cabo para sua casa.
Os telefones celulares usam ondas de rádio
para transmitir informação. Estas ondas são muito
menores que as da TELEVISÃO e ondas de rádio de
FM.
Por que as antenas estão em telefones
celulares são menores que antenas em seu radio?
Como nós ―vemos‖ usando ondas de rádio?
Objetos no espaço, como planetas e cometas,
nuvens gigantes de gás, estrelas e galáxias, emitem luz
a muitos comprimentos de onda diferentes. Algumas
das luzes que eles emitem tem comprimentos de onda
muito grandes - às vezes quase que um milhão. Estas
ondas longas estão na região de rádio do espectro
eletromagnético.
As ondas de rádio são maiores que ondas
ópticas, e as antenas que captam ondas de rádio
trabalham diferentemente que telescópios que nós
usamos para luz visível (telescópios ópticos). Radio
telescópios são extensas superfícies parabólicas de
metal que refletem ondas de rádio para um ponto
focal. Devido os comprimentos de onda de luz de
rádio serem tão grandes, radiotelescópios devem ser
fisicamente maiores que um telescópio óptico para que
possa fazer imagens de claridade comparável. Por
exemplo, o Parkes rádio telescópio tem uma
circunferência de raio de 64 metros.
Para fazer imagens de rádio melhores (ou
resolução mais alta) o rádio astrônomos combina
freqüentemente vários telescópios menores, ou pratos
receptores, em uma certa ordem. Juntos, os pratos
podem agir como um telescópio grande cujo tamanho
se iguala à área total ocupado pelo conjunto.
O VLA é um dos primeiros observatórios
rádios astronômicos do mundo. O VLA consiste em 27
antenas organizadas em forma de Y ―enorme‖ de até
36 km (22 milhas) uma vez e meias o tamanho de
Washington, DC.
O VLA, localizado em Novo México, é um
interferômetro; isto significa que opera multiplicando
os dados junto de cada par de telescópios para formar
padrões de interferência. A estrutura desses padrões de
interferência, e como eles mudam com o tempo
conforme a Terra gira refletem a estrutura de fontes de
rádio no céu. O que nos mostram as ondas de rádio?
Muitos objetos astronômicos emitem ondas
de rádio, fato que não foi descoberto até 1932. Desde
então,
astrônomos
desenvolveram
sistemas
sofisticados que lhes permitem fazer fotografias das
ondas de rádio emitidas por objetos astronômicos. As
ondas de Rádio provenientes dos céus são devido a
planetas e cometas, nuvens gigantes de gás e poeira,
estrelas e galáxias. Estudando as ondas de rádio
originadas destas fontes, astrônomos podem aprender
sobre a composição delas, sua estrutura e movimento.
A Radio Astronomia tem a vantagem que a luz solar,
nuvens, e chuva não afetam as observações.

Radiação infravermelha:
Medidas de luz infravermelha estão
compreendidas entre o visível e as microondas do
espectro eletromagnético. Luz infravermelha tem um
alcance de comprimentos de onda que variam
próximos da luz vermelha a violeta. "Luz
infravermelha próxima" é comparável em
comprimento de onda para luz visível e
"infravermelho longínguo" é próximo à região de
microondas do espectro eletromagnético. Os
comprimentos de onda infravermelhos mais longos
estão próximos ao tamanho de uma cabeça de alfinete
e o infravermelho próximo é microscópico.
Ondas infravermelhas distantes são térmicas.
Em outra palavra, nós experimentamos este tipo de
radiação infravermelha diariamente na forma de
calor! O calor que nós sentimos de luz solar, um
fogo, um radiador ou uma calçada morna é
infravermelha. Os terminais nervosos em nossa pele
possuem sensibilidade para poder descobrir a
diferença entre a temperatura do corpo à temperatura
da pele.
Usamos a radiação infravermelha para aquecer
até mesmo a comida; luminárias especiais que
emitem ondas infravermelhas térmicas são
freqüentemente usadas em restaurantes de comida
rápidas (fast food).
Ondas infra-vermelhas menores, próximas, não
são quentes, de fato você nem mesmo as sente. Estes
comprimentos de onda menores são os usados por
seu o controle remoto de Televisão.
Como nós ―podemos ver‖ usando o
Infravermelho?
Como a fonte primária de radiação
A figura acima mostra espetáculos da imagem infravermelha é gerada pelo calor ou radiação
de Monóxido de Carbono (CO) e gases em nossa térmica, qualquer objeto numa temperatura radia no
galáxia Via Láctea.
infravermelho. Até mesmo objetos que nós pensamos
estarem muito frios, como um cubo de gelo, emite
31
Física Moderna – Capítulo 4 - Radiação de corpo negro e Efeito Fotoelétrico
Prof. Dr. Cláudio S. Sartori
infravermelho. Quando um objeto não está bastante
quente para radiar luz visível, emitirá a maioria de sua
energia no infravermelho. Por exemplo, carvão quente
pode não emitir luz, mas emite radiação infravermelha
e nós sentimos como calor. Até o mais morno objeto
emite radiação infravermelha.
Humanos, a temperatura de corpo normal, radiam
fortemente no infravermelho a um comprimento de
Para uma visão da órbita de Terra, se nós estamos
olhando fora em espaço ou descemos em Terra, nós
podemos usar instrumentos a bordo de satélites.
Satélites como o VAI 6 e Landsat 7
observam a Terra com sensores especiais, como esses
a bordo o Landsat 7 satélite, dados de registro sobre a
quantidade de luz infra-vermelha refletida ou emitida
da superfície da Terra.
onda de cerca de 10 mícrons. 1m  10 m (Um
mícron é o termo comumente usado em física para um
micrômetro ou um milionésimo de um metro)
6
32
Esta imagem (que é cortesia do Processamento
Infra-vermelho e Centro de Análise do CalTech),
ilustra como é a fotografia em infravermelho de um
homem que sustenta uma pinça!
Qual objeto desta imagem você acha ter a
temperatura mais morna?
Como a temperatura dos óculos deste homem se
compara à temperatura da mão dele?
Para fazer
fotografias
infravermelhas
similares à de cima, podemos usar máquinas
fotográficas especiais e filmes que possuem diferenças
em temperatura, e então colocar diferentes ou falsas
cores a eles. Isto resulta numa fotografia que nossos
olhos podem interpretar.
A imagem (cortesia da Corporação de SE-IR,
Goleta, a CA) é uma fotografia de um gato no infravermelho. As áreas laranja são os mais mornos e as
áreas branco-azuis são os mais frios. Esta imagem nos
dá uma visão diferente de um animal familiar como
também informação que nós não pudéssemos obter de
uma
fotografia visível.
Humanos não podem ver luz infravermelha,
mas você sabia que serpentes, como víboras ou
cascavéis, têm órgão "sensoriais‖ que são usados para
detectar a imagem de luz infravermelha? Isto permite
que a serpente descubra animais de sangue morno, até
mesmo em covas escuras! Algumas serpentes possuem
órgãos sensoriais com até mesmo percepção de
profundidade no infravermelho!
Muitas coisas além de pessoas e animais
emitem luz infravermelha - a Terra, o Sol, objetos
distantes como estrelas e galáxias também o fazem!
Outros satélites, como o Satélite de
Astronomia Infra-vermelho (IRAS) observam do
espaço e medem a radiação infravermelha nuvens
grandes de pó e gás que podem formar estrelas,e
galáxias!
O que o infravermelho nos mostra?
Esta é uma imagem infravermelha da Terra
tirada pelo satélite VAI 6 em 1986. Cientistas
observam temperaturas diferentes para determinar
quais partes da imagem são de nuvens, terra e mar.
Baseado nestas diferenças de temperatura, usando
256 cores separadamente, coloriu-se a imagem
tornando uma fotografia realista.
Por que usamos o infravermelho para tirar
uma fotografia da Terra? Enquanto é mais fácil de
distinguir nuvens da Terra no alcance visível, há mais
detalhes no interior das nuvens no infra-vermelho.
Pode-se estudar a estrutura da nuvem.
Por exemplo, nota-se que as nuvens mais
escuras estão mais mornas, enquanto nuvens mais
claras estão mais frescas.
No sudeste do Galápagos, só no oeste da
costa de América do Sul, há um lugar onde você
32
Física Moderna – Capítulo 4 - Radiação de corpo negro e Efeito Fotoelétrico
Prof. Dr. Cláudio S. Sartori
33
pode ver capas múltiplas de nuvens distintamente,
com as nuvens mais mornas a mais baixas altitudes,
mais próximo ao oceano que está esquentando.
Nós sabemos, ao olhar umas imagens
infravermelhas de um gato, que muitas coisas emitem
luz infravermelha. Mas muitos objetos também
refletem luz infravermelha, particularmente luz
infravermelha próxima. A radiação infravermelha
próxima não é relacionada à temperatura do objeto que
é fotografado - a menos que o objeto seja muito, muito
quente.
Um filme infravermelho ―fotografa‖ o objeto
porque o Sol (ou alguma outra fonte clara) forneceu
luz infravermelha no filme e é refletido ou absorvido
pelo objeto. Você poderia dizer que refletindo ou
absorvendo infravermelho ajuda a determinar a cor do
objeto - sua cor sendo uma combinação de vermelho,
verde, azule, e infravermelho!
Estudos mostram que a clorofila em plantas
reflete ondas infravermelhas próximas junto com
ondas de luz visíveis. Embora nós não possamos ver as
ondas infravermelhas, eles sempre estão lá.
Outro fenômeno importante relacionada com
a radiação infravermelha é o efeito estufa.
O aumento do gás CO2 pode produzir maior
retenção dessa radiação infravermelha produzida pela
Terra, superaquecendo o Planeta.
Instrumentos a bordo de satélites também
podem tirar fotos de objetos no espaço. A imagem
debaixo da região do centro de nossa galáxia foi tirada
pelo satélite IRAS.

Radiação Visível:
Ondas claras visíveis são as únicas ondas
eletromagnéticas que nós podemos ver. Nós vemos
estas ondas como as cores do arco-íris. Cada cor tem
um comprimento de onda diferente. Vermelho tem o
comprimento de onda mais longo e violeta tem o
comprimento de onda menor. Quando todas as ondas
são vistas juntas, eles formam a luz branca.
Quando um raio de luz branca passa por um
prisma ou por vapor de água como este arco-íris, a
luz branca separa-se nas cores do espectro claro
visível.
Como nós ―vemos‖ usando Luz Visível?
Os cones em nossos olhos são os receptores
A característica da nebulosa em forma de S, para estas ondas de luz visíveis minúsculas. O Sol é
horizontal que cruza a imagem é o calor emitido pelas uma fonte natural para ondas de luz visíveis e nossos
nuvens de poeira do sistema solar.
olhos observam a reflexão desta luz solar dos objetos
ao nosso redor.
A cor de um objeto que nós vemos é a cor de
luz refletida. Todas as outras cores são absorvidas.
Lâmpadas incandescentes são outra fonte de
ondas de luz visíveis.
Estas são imagens de Phoenix, Arizona, uma
fotografada por uma nave e outra colorida. Você
pode ver uma diferença entre esta imagem e a
fotografia a seguir?
33
Física Moderna – Capítulo 4 - Radiação de corpo negro e Efeito Fotoelétrico
Prof. Dr. Cláudio S. Sartori
nós usamos satélites para olhar a Terra, e telescópios
para olhar o Céu!
Nós não só olhamos a Terra do espaço, mas
nós também podemos olhar outros planetas de
espaço. Esta é uma imagem clara visível do planeta
Júpiter. Está em falsa cor - as cores foram escolhidas
para enfatizar a estrutura de nuvem que atua no
planeta Júpiter e não apareceria a seus olhos.

34
Radiação ultravioleta (UV):
Há dois tipos de imagens coloridas que
podem ser feitas de dados de satélite – as de cores
verdadeiras e as coloridas artificialmente. Tirar
imagens coloridas, como esta aqui, o satélite que tirou
isto usou um sensor para registrar dados sobre as
ondas de luz visíveis vermelhas, verdes, e azuis que
estavam refletindo a superfície da terra. Os dados
foram combinados num computador mais tarde. O
resultado é semelhante ao que nossos olhos vêem.
Uma imagem de cor falsa é feita quando o
satélite registra dados sobre brilho das ondas claras
que refletem a superfície da Terra. Estes brilhos são
representados por valores numéricos - e estes valores
podem ser codificados por cores. É como pintar
através de números! As cores escolhidas que pintam a
imagem são arbitrárias, mas eles podem ser escolhidos
ou fazer o objeto parecer realista, ou ajudar a enfatizar
uma característica particular na imagem. Astrônomos
podem ver uma região de interesse até mesmo usando
software para mudar o contraste e brilho no quadro,
como os controles em uma TELEVISÃO! Você pode
ver uma diferença nas paletas de cor selecionadas para
as duas imagens abaixo?
Ambas as imagens são da Nebulosa de
Caranguejo, os restos de uma estrela explodida!
A luz ultravioleta (UV) tem comprimentos
de onda menores que luz visível. Embora estas ondas
sejam invisíveis ao olho humano, alguns insetos,
como abelhas, os podem ver.
Cientistas dividiram a parte ultravioleta do
espectro em três regiões: o ultravioleta próximo, o
ultravioleta distante, e o ultravioleta extremo. As três
regiões são distintas pela energia da radiação
ultravioleta e pelo comprimento de onda da luz
ultravioleta que é relacionada com a energia.
O NUV ultravioleta próximo, abreviado por
NUV – Near Ultra-violet, é a luz mais próxima da luz
óptica ou visível. O ultravioleta extremo, abreviado
EUV, é a luz ultravioleta mais próxima para
Radiografias, e é o mais enérgico dos três tipos. O
ultravioleta distante, abreviado FUV, encontra-se
entre as próximas e extremas regiões ultravioletas. É
o menos explorado das três regiões.
Nosso Sol emite luz a todos os
comprimentos de onda diferentes em espectro
eletromagnético, mas são as ondas ultravioletas que
são responsáveis para causar nossas queimaduras de
sol.
O que nos mostra Luz Visível?
É verdade que nós somos cegos a muitos
comprimentos de onda de luz. Por isso usamos
instrumentos que podem descobrir comprimentos de
onda diferentes de luz para nos ajudar a estudar a
Terra e o Universo. Porém, desde que luz visível é
parte do espectro eletromagnético que nossos olhos
podem ver, nosso mundo inteiro é orientado ao redor
disso. E muitos instrumentos que descobrem luz
visível podem ver mais claramente que nossos olhos,
com maior sensibilidade à radiação. Por isso é por que
À esquerda é uma imagem do Sol tirada a
um comprimento de onda Ultravioleta Extremo - 171
Angstroms para ser exato. (Um Angstrom é igual a
10-10 metros.) Esta imagem foi tirada por um satélite
denominado SOHO e mostra o Sol em 24 de abril de
2000.
Embora algumas ondas ultravioletas do Sol
penetrem a atmosfera de Terra, a maioria delas são
bloqueadas ao penetrá-la por vários gases como o
Ozônio (O3).
34
Física Moderna – Capítulo 4 - Radiação de corpo negro e Efeito Fotoelétrico
Prof. Dr. Cláudio S. Sartori
35
Cientistas desenvolveram um índice de UV
para ajudar as pessoas a se proteger destas ondas
prejudiciais.
Como nós ―vemos‖ usando luz Ultravioleta?
É bom para nós que somos humanos estar
protegido de adquirir muita radiação ultravioleta, mas
é ruim para os cientistas! Astrônomos têm que colocar
telescópios ultravioleta em satélites e medir a luz
ultravioleta de estrelas e galáxias - e coisas até mais
próximas como o Sol!
Há muitos satélites diferentes que nos ajudam
e estudam a astronomia ultravioleta. Muitos deles só
descobrem uma pequena quantidade de luz UV. Por
exemplo, o telescópio espacial Hubble observa estrelas
e galáxias principalmente em luz ultravioleta próxima.
O satélite Explorador Ultravioleta Extremo da NASA
está explorando o universo ultravioleta extremo
atualmente. O satélite Explorador Ultravioleta
Internacional (IUE) observou as regiões ultravioletas
distantes e próximas durante mais de 17 anos.
O que nos mostra luz Ultravioleta?
Nós podemos estudar estrelas e galáxias
estudando o UV que elas emitem - mas você sabia que
nós podemos estudar até mesmo a Terra?
O UV Camera/Spectrograph Distante tirou a
foto a seguir. A parte da Terra iluminada pelo Sol
reflete muita luz UV.
Aqui, faixas de emissão de UV também são
aparentes. Estas faixas são o resultado de aurora
causada por partículas carregadas emitidas pelo Sol.
Elas espiralam para a Terra ao longo das linhas de
campo magnético da Terra.
Muitos cientistas estão interessados em
estudar o universo invisível de luz ultravioleta, desde
os mais quentes e os objetos mais ativos no cosmo que
emitam quantias grandes de energia ultravioleta.
A imagem ao lado ilustra três galáxias
diferentes obtidas em luz visível (fundo três imagens)
e luz ultravioleta (fila de cima) tirada pelo Imaging
Telescope Ultravioleta da NASA (UIT) na missão
Astro-2.
As diferenças de como as galáxias aparecem é
devido ao tipo de brilho de estrelas mais luminosas nos
comprimentos de onda ópticos e ultravioletas. As
fotografias dessas galáxias indicam principalmente
nuvens de gás contendo que formarão estrelas
recentes muitas vezes mais volumosas que o sol que
arde fortemente em luz ultravioleta. Em contraste,
fotografias de luz visíveis de galáxias principalmente
a luz amarela e vermelha são de estrelas mais velhas.
Comparando estes tipos de dados, astrônomos podem
aprender sobre a estrutura e evolução de galáxias.
 Radiação X (Raios X):
Com a diminuição dos comprimentos de
onda, eles aumentam sua energia. Radiografias têm
comprimentos de onda menores e então energia mais
alta que ondas ultravioletas. Nós normalmente
falamos sobre Radiografias em termos da energia em
lugar de comprimento de onda. Isto porque
Radiografias têm comprimentos de onda muito
pequenos. Também é porque luz de Radiografia
tende a agir mais como uma partícula que uma onda.
Detectores de radiografia absorvem fótons de luz de
Raios X - que é muito diferente dos telescópios de
rádio que têm pratos grandes projetados para detectar
ondas de rádio!
Foram observados as primeiras radiografias
e documentadas em 1895 por Wilhelm Conrad
Röentgen, um cientista alemão que a descobriu
acidentalmente
quando
estava
realizando
experiências com tubos de vácuo.
Uma semana depois, ele levou uma
fotografia de Radiografia da mão de sua esposa que
claramente revelou o anel de casamento dela e seus
ossos. A fotografia assombrou o público em geral e
grande interesse científico foi despertado nessa nova
forma de radiação. Röentgen chamou isto de radiação
―X‖ para indicar que era um tipo desconhecido de
radiação. O nome aderiu, embora (em cima das
objeções de Röentgen), muitos dos seus colegas
questionaram os chamando raios de Röentgen. Eles
ainda são ocasionalmente chamado raios de Röentgen
em países de língua alemã.
A atmosfera da Terra é espessa bastante que
virtualmente nenhuma radiação na faixa dos Raios X
pode penetrar do espaço exterior para a superfície da
Terra. Isto é bom para nós, mas também ruim para
astronomia - temos que pôr telescópios e detectores
de Raios X em satélites! Nós não podemos fazer
astronomia de raios X do solo.
Como nós ―vemos‖ usando os Raios X?
Bem, nós não poderíamos ver pelas roupas
de pessoas, não importa o que os anúncios para
óculos de Raio X nos contam! Se nós pudéssemos
ver os Raios X, nós poderíamos ver coisas que ou
emitem Raio X ou detêm a transmissão deles. Nossos
olhos estariam como o filme de Radiografia usado
em hospitais ou os escritórios de dentista. Filme de
radiografia ―vê‖ Raios X.
35
Física Moderna – Capítulo 4 - Radiação de corpo negro e Efeito Fotoelétrico
Prof. Dr. Cláudio S. Sartori
36
Quando você adquire uma Radiografia tirada
em um hospital, um filme sensível é posto em um lado
de seu corpo, e são atiradas Radiografias sobre você.
Num consultório dentário, o filme é posto dentro de
sua boca, em um lado de seus dentes, e são atiradas
Radiografias por sua mandíbula. Não dói nada - você
não pode sentir Raio X.
É porque seus ossos e dentes são densos e
absorvem mais raio X que sua pele, produz-se
silhuetas de seus ossos ou dentes que permanecem no
filme de Radiografia enquanto sua pele aparece
transparente. Metais absorvem mais Raios X - você
pode ver o objeto metálico na imagem do dente?
Quando o Sol nos ilumina num certo ângulo,
nossa sombra é projetada sobre o solo.
Semelhantemente, quando os Raios X incidem em nós,
passa por nossa pele, mas permite projetar sombras
sobre nossos ossos e são capturadas através de um
filme.
Abaixo vemos a fotografia de Radiografia de
uma menina. Você pode ver a sombra do objeto que
ela engoliu?
Ao centro vemos a Radiografia da mão da
esposa de Röentgen.
sistemas binários de estrelas, sobras de supernova,
estrelas, o Sol, e até mesmo alguns cometas!
A Terra emite muitos tipos de luz, inclusive
a faixa de Raios-X enérgica. De fato, a própria Terra
emite - a aurora produz na atmosfera da Terra. Esta
aurora é causada pela incidência na atmosfera de
partículas carregadas do Sol.
A foto é do satélite Polar, PIXIE, NASA e à
esquerda está a primeira fotografia da Terra em
Raios-X, tirada em março de 1996 com o satélite
Polar orbital. A área de emissão de Radiografia mais
luminosa é vermelha. As partículas carregadas
enérgicas do Sol que causam a aurora também
energizam elétrons na magnetosfera da Terra. Estes
elétrons movem sobre o campo magnético da Terra e
eventualmente golpeiam as moléculas da ionosfera da
Terra e causam a emissão de Raios X. Estes Raios X
não são perigosos porque eles são absorvidos por
partes mais baixas da atmosfera da Terra.
Recentemente, nós aprendemos que cometas
emitem Radio X! A imagem acima do Cometa
Hyakutake foi tirada por um satélite de Raios-X
chamado ROSAT.
O Sol também emite Raios X – a foto da
direita é do Sol observado por Radiografia de 27 de
abril de 2000. Esta imagem foi tirada pelo satélite
Yokoh.
Muitas estrelas formam sistemas binários –
duas estrelas que orbitam uma em relação à outra.
Quando uma destas estrelas é um buraco negro ou
uma estrela de nêutrons, material é puxado da estrela
normal. Estas espirais de materiais no buraco negro
Nós usamos satélites com detectores de ou na estrela de nêutron possuem temperaturas muito
Raios-X para Radiografar imagens em astronomia. Em altas. Quando algo é aquecido a mais de um milhão
astronomia, objetos que emitem Raios-X (por de graus, emitirá Raios X!
exemplo, buracos negros) são como uma máquina de
Radiografia do dentista, e o detector no satélite
funciona como o filme de Radiografia. Detectores de
raios-X absorvem Raios individuais (fótons de luz de
Raios-X) e o número de fótons coletados, a energia
dos fótons, ou quão rápidos os fótons são absorvidos,
podem nos contar informações importantes sobre o
objeto que os está emitindo. À direita há uma imagem
A imagem esquerda anterior é a concepção
de um detector de Raio X. Este
de um artista de um sistema de estrela binário e
instrumento está no satélite Explorador (RXTE). mostra o material sendo puxado da estrela vermelha
Parece muito diferente de qualquer coisa que você por seu companheiro, um buraco negro invisível e em
poderia ver no escritório de um dentista!
um disco de órbita. A imagem à direita mostra uma
O que os Raios X nos mostra?
sobra de supernova - a sobra de uma estrela que
Muitas coisas no espaço emitem Raios-X, explodiu em uma galáxia perto conhecida como a
entre eles estão buracos negros, estrelas de nêutrons, Nuvem de Magalhães Pequena. As colorizações
36
Física Moderna – Capítulo 4 - Radiação de corpo negro e Efeito Fotoelétrico
Prof. Dr. Cláudio S. Sartori
37
utilizadas mostram que esta sobra de supernova seriam substituídas por algo sempre variável. Sua
emitem em Raio X (em azul), luz visível (verde) e de visão de raio gama apareceria labaredas solares,
rádio (vermelho).
supernovas, estrelas de nêutons, buracos negros, e
galáxias ativas. Astronomia de raios gama
apresentam oportunidades sem igual para explorar
 Radiação Gama (Raios ):
estes objetos exóticos. Explorando o universo a estas
Os Raios Gama têm os comprimentos de onda altas energias, cientistas podem procurar uma nova
menores e a maior energia de qualquer outra onda no física, testar teorias e executar experiências que não
espectro eletromagnético. Estas ondas são geradas são possíveis em laboratórios da Terra.
Se você pudesse ver raios raios gama,
através de átomos radioativos e em explosões
estrelas de nêutrons ou pulsares estariam entre os
nucleares.
Raios Gama podem matar células vivas, um objetos mais luminosos no céu. Este computador
fato que a medicina utiliza para matar células processou imagem que mostra o pulsar de Nebulosa
de Caranguejo (debaixo de e a direito do centro) e o
cancerosas.
pulsar de Geminga (sobre e a partir do
Os raios Gama viajam a nós por distâncias
vastas do universo e são absorvido pela atmosfera da centro) na "faixa da luz" de raios gama raios.
Terra. Comprimentos de onda diferentes de luz
penetram a atmosfera da Terra para profundidades
diferentes. Instrumentos a bordo balões de alta-altitude
e satélites como o Observatório de Compton provêem
nossa única visão do céu de gama-raio.
Raios Gama são a forma mais enérgica de luz
e são produzidos pelas regiões mais quentes do
universo. Eles também são produzidos através de
eventos violentos como explosões de supernova ou a
destruição de átomos, e através de eventos como o
decaimento de material radioativo no espaço. Coisas
como explosões de supernova (o modo como as
estrelas volumosas morrem), estrelas de nêutrons,
pulsares e buracos negros são todas as fontes celestiais
de raios gama.
Como nós ―vemos‖ usando luz de raios-gama?
A nebulosa de Caranguejo, também
A Astronomia de raio-gama não se desenvolveu
mostrada
na imagem clara visível, foi criada por uma
até que fosse possível colocar detectores acima da
atmosfera, usando balões ou satélites. O primeiro supernova que clareou o céu noturno em 1054 D.C.
telescópio de raios gama, levado em órbita pelo Em 1967, astrônomos descobriram o caroço que
satélite Explorador XI em 1961, capturou menos que sobrou daquela estrela; um rápido pulsar giratório,
magnético que produz ondas de rádio a cada 0.33
100 fótons de raios gama cósmicos!
Luz óptica distinta e Radiografias não podem ser segundos.
Talvez a descoberta mais espetacular em
utilizadas para capturar raios gama e podem ser
refletidos em espelhos. Os fótons de alta-energia astronomia de raios gama ocorreu nos anos 1960s e
passariam direto por tal dispositivo. Para detectar raios setenta. Um Detector a bordo do satélite Vela,
Gama usa-se um processo chamado Compton, onde satélites originalmente militares, começou a registrar
um fóton de raio gama golpeia um elétron e perde estouros de gama-raios não da Terra, mas do espaço
energia, semelhante a uma bola que golpeia uma outra profundo!
Explosões de raio gama podem lançar mais
bola.
energia em 10 segundos que o Sol emitirá em sua
vida inteira de 10 bilhões de anos! Tão longe, aparece
que os estouros que nós observamos vieram de fora
da Galáxia da Via Láctea. Os cientistas acreditam
que as explosões de raio gama acontecem a alguns
milhões de anos na Via Láctea, e de fato pode
acontecer uma vez a cada cem milhões de anos e
O que nos mostram os raios gama?
Se você pudesse ver a radiação gama, o céu dentro de alguns mil ano-luz da Terra.
Estudado agora durante mais de 25 anos
noturno pareceria estranho e pouco conhecido.
A lua de vista por meio de raios gama emitida com instrumentos a bordo de uma variedade de
por ela apareceria como uma redonda gota - satélites e sondas de espaço, inclusive astronave de
características lunares não seriam visíveis. Em raios de Venera soviética e o Vênus Orbiter Pioneiro, as
gama de alto-energia, a Lua é realmente mais luminosa fontes destes flash de alto-energia enigmáticos
que o Sol. As visões familiares de estrelas e galáxias permanecem um mistério.
37
Física Moderna – Capítulo 4 - Radiação de corpo negro e Efeito Fotoelétrico
Prof. Dr. Cláudio S. Sartori
Resolvendo o mistério de estouros de gamaraio, cientistas esperam ganhar conhecimento
adicional das origens do Universo, a taxa à qual o
Universo está se expandindo, e o tamanho do
Universo.

38
Microondas
As Microondas têm comprimentos de onda
que podem ser medidos em centímetros! A microonda
mais longa, esses mais próximo de um pé, são as
ondas que aquecem nossa comida em um forno de
microondas.
Microondas são bons para transmitir
informação de um lugar para outro porque energia da
microonda pode penetrar névoa, chuva clara e neva,
nuvens, e fumaça.
Microondas menores são usados sentindo
distante. Esta microonda é usada para radar como o
radar Doppler usado em previsões de tempo.
A torre de microondas pode transmitir
informações para telefonia celular e dados de
computador de uma cidade para outra.
Transmissão da
Freqüências
(MHz)
Estação
Móvel
ERB
Subfaixa
A**
824-835
845-846,5
869-880
890-891,5
Subfaixa
B**
835-845
846,5-849
880-890
891,5-894
Subfaixa D
910-912,5
1710-1725
955-957,5
1805-1820
Subfaixa E
912,5-915
1740-1755
957,5-960
1835-1850
Subfaixas
de Extensão
898,5-901*
907,5-910*
1725-1740
1775-1785
943,5-946*
952,5-955*
1820-1835
1870-1880
* Não serão autorizadas para prestadoras do SMP operando nas
Bandas D e E. Todas as operadoras de Banda D e E adquiriram
também as faixas de frequências de 900 MHz alocadas para a sua
Banda.
** Admite o emprego de sistemas analógicos (AMPS) nas Bandas
A e B até 30/06/2008.
Novas Bandas do SMP
Res. 454 de 11/12/06 que revogou a Res. 376 02/09/04.
MHz
Transmissão da
Subfaixa
Estação
Móvel
ERB
F*
1920-1935
2.1102.125
G*
1.935-1.945
2.1252.135
H*
1.945-1.955
2.1352.145
I*
1.955-1.965
2.1452.155
Os radares foram desenvolvidos para
2.155descobrir objetos e determinar o alcance deles (ou
J*
1.965-1.975
2.165
posição) transmitindo pequenos sinais de microondas.
São registradas a força e origem de "ecos" recebidas
1.975L
1.895-1.900
1.980
de objetos que foram rebatidos pelas microondas.
O radar detecta ondas eletromagnéticas que
1.850M
1.755-1.765
são uma reflexão de uma transmissão ativa, e é
1.860
considerado um sistema distante ativo. Sistema
1.860distante passivo se refere ao sentindo de ondas
1.765-1.770
1.865
eletromagnéticas que não originaram do satélite ou
1.770-1.775
1.865Subfaixa
de
sensor. O sensor é um observador passivo.
1.870
Extensão
 Bandas de Frequências para o Celular
1.885-1.890**
no Brasil Estão disponíveis para o celular no Brasil
1.890-1.895**
(SMP) frequências nas bandas de:
* Faixas reservadas para sistemas 3G
 850 MHz, antigas bandas A e B
** Sistemas TDD (Time Division Duplex) que utilizam a mesma
 900 MHz, bandas de extensão utilizadas pelo subfaixa de frequências para transmissão nas duas direções.
GSM.
 1700 e 1800 MHz, bandas D, E e subfaixas de
extensão utilizadas pelo GSM
 1900 e 2100 MHZ destinadas na sua maior
parte para sistemas 3G.

38
Física Moderna – Capítulo 4 - Radiação de corpo negro e Efeito Fotoelétrico
Prof. Dr. Cláudio S. Sartori
39
http://www.teleco.com.br/Bandac.asp
Como as microondas podem penetrar névoa,
chuva clara e neva, nuvens e fuma, estas ondas são
boas para ver a Terra do espaço.
O ERS-1 satélite envia comprimentos de onda
aproximadamente 5.7 cm (faixa-C).
O satélite de JERS usa comprimentos de
onda aproximadamente 20 cm em duração (faixa L).
Nos anos de 1960 uma descoberta
surpreendente foi feita totalmente através de acidente.
Um par de cientistas em Laboratórios de Sino
descobriu ruído de fundo utilizando uma antena de
rádio especial. A coisa estranha sobre o ruído era que
estava vindo de toda direção e não parecia variar em
intensidade muito nada. Se esta fosse estática de algo
em nosso mundo, iguais transmissões de rádio de um
aeroporto perto que controlava a torre, só viria de uma
direção, não em todos lugares. Os cientistas
perceberam logo que eles tinham descoberto a
radiação de fundo de microonda cósmica. É acreditado
que esta radiação que enche o Universo inteiro é
devida ao conhecido Big Bang.
39
Física Moderna – Capítulo 4 - Radiação de corpo negro e Efeito Fotoelétrico
Prof. Dr. Cláudio S. Sartori
40
40
Física Moderna – Capítulo 4 - Radiação de corpo negro e Efeito Fotoelétrico
Prof. Dr. Cláudio S. Sartori

Apêndice

Espectros de estrelas
(Adaptado de:
http://docs.kde.org/stable/pt_BR/kdeedu/kstars/aicolorandtemp.html)
41
As estrelas parecem ser exclusivamente
brancas a primeira vista. Mas se olharmos
cuidadosamente, podemos notar uma faixa de cores:
azul, branco, vermelho e até dourado. Na constelação
de Orion, um bonito contraste é visto entre o vermelho
de Betelgeuse no "sovaco" de Orion e o azul de
Bellatrix no ombro. O que faz estrelas exibirem cores
diferentes permanecia um mistério até dois séculos
atrás,
quando
físicos
obtiveram
suficiente
conhecimento da natureza da luz e propriedades da
matéria em temperaturas imensamente altas.
Especificamente, foi a física da radiação dos
corpos negros que nos possibilitou entender a variação
das cores estelares. Logo após o entendimento do que
era a radiação dos corpos negros, notou-se que o
espectro das estrelas parecia extremamente similar as
curvas da radiação dos corpos negros em várias
temperaturas, variando de poucos milhares de Kelvin
até 50.000 Kelvin. A conclusão óbvia é que estrelas
são semelhantes a corpos negros, e que a variação de
cor das estrelas é uma consequência direta da
temperatura de sua superfície.
Estrelas frias (isto é, Espectro Tipo K e M)
irradiam a maior parte de sua energia na região
vermelha
e
infravermelha
do
espectro
electromagnético e assim parecem vermelhas,
enquanto estrelas quentes (isto é, Espectro Tipo O e B)
emitem principalmente em comprimentos de onda azul
e ultravioleta, fazendo-as parecerem azul ou brancas.
Para estimar a temperatura superficial de uma
estrela, podemos usar a conhecida relação entre
temperatura de um corpo negro e o comprimento de
onda da luz no pico de seu espectro. Isto é, conforme
você aumenta a temperatura de um corpo negro, o pico
de seu espectro move-se para um menor (mais azul)
comprimento de onda luminoso. Isto é ilustrado na
Figura 1 abaixo onde a intensidade de três estrelas
hipotéticas é plotada contra o comprimento de onda. O
"arco-íris" indica a faixa de comprimento de onda que
é visível ao olho humano.
Figura 1 – Espectro de estrelas de diferentes cores.
Este método simples é conceitualmente
correto, mas não pode ser usado para obter
temperaturas
estelares
precisas,
porque
estrelas não são corpos negros perfeitos. A presença
de vários elementos na atmosfera estelar fará com
que alguns comprimentos de onda sejam absorvidos.
Devido a estas linhas de absorção não serem
uniformemente distribuídas no espectro, elas podem
inclinar a posição do pico espectral. Além disso,
obter um espectro estelar é um processo de tempo
intensivo e é proibitivamente difícil para grandes
amostras de estrelas.
Um método alternativo utiliza a fotometria
para medir a intensidade da luz passando por
diferentes filtros. Cada filtro permite apenas uma
parte específica do espectro passar enquanto todas as
outras são rejeitadas. Um sistema fotométrico muito
utilizado chama-se sistema UBV Johnson. Ele
emprega três filtros de banda: U ("Ultra-violeta"), B
("Azul"), and V ("Visível"), cada uma ocupando as
diferentes regiões do espectro eletromagnético.
O processo de fotometria UBV envolve usar
dispositivos foto sensíveis (como filmes ou câmeras
CCD) e mirar um telescópio em uma estrela para
medir a intensidade da luz que passa por cada filtro
individualmente. Este processo fornece três
luminosidades aparentes ou fluxos (quantidade de
energia por cm2 por segundo) designados por Fu, Fb
e FV. A relação dos fluxos Fu/Fb e Fb/Fv é uma
medida quantitativa da "cor" da estrela, e estas
relações podem ser usadas para estabelecer uma
escala de temperatura para estrelas. Falando
genericamente, quanto maiores as relações Fu/Fb e
Fb/Fv de uma estrela, mais quente é sua temperatura
de superfície.
Por exemplo, a estrela Bellatrix em Orion
tem um Fb/Fv = 1,22, indicando que é mais brilhante
pelo filtro B que pelo filtro V. Além disso, sua razão
Fu/Fb é 2,22, então é mais brilhante pelo filtro U.
Isto indica que a estrela deve ser muito quente
mesmo, pois seu pico espectral deve estar em algum
lugar na faixa do filtro U, ou até mesmo em
comprimentos de onda mais baixos. A temperatura
superficial de Bellatrix (determinada por comparação
de seu espectro com modelos detalhados que
conferem com suas linhas de absorção) é perto de
25.000 Kelvin.
Podemos repetir esta análise para a estrela
Betelgeuse. Suas razões Fb/Fv e Fu/Fb são 0.15 e
0.18 respectivamente, então ela é mais brilhante em
V e mais opaca em U. Então, o pico espectral de
Betelgeuse deve estar em algum lugar na faixa do
filtro V, ou mesmo em um comprimento de onda
superior. A temperatura superficial de Betelgeuse é
de apenas 2,400 Kelvin.
Os astrônomos preferem expressar as cores
estelares em termos de diferença em magnitudes, do
que uma razão de fluxos. Assim, voltando para a azul
Bellatrix temos um índice de cor igual a
B - V = -2.5 log (Fb/Fv) = -2.5 log (1.22) = -0.22,
41
Física Moderna – Capítulo 4 - Radiação de corpo negro e Efeito Fotoelétrico
Prof. Dr. Cláudio S. Sartori
Similarmente, o índice de cor para a vermelha
Betelgeuse é
B - V = -2.5 log (Fb/Fv) = -2.5 log (0.18) = 1.85
Os índices de cores, como a escala de
magnitude ,correm para trás. Estrelas Quentes e
azuis têm valores de B-V menores e negativos que as
mais frias e vermelhas estrelas.
Um Astrônomo pode então usar os índices de
cores para uma estrela, após corrigir o
avermelhamento e extinção interestelar, para obter
uma precisa temperatura daquela estrela. A relação
entre B-V e temperatura é ilustrada na Figura 2.
Figura 2 – Relação B-V e temperatura.
42

Pirômetros
Um pirómetro (também denominado de
pirómetro óptico) é um dispositivo que mede
temperatura sem contacto com o corpo/meio do qual
se pretende conhecer a temperatura. Geralmente este
termo é aplicado a instrumentos que medem
temperaturas superiores a 600 graus celsius. Uma
utilização típica é a medição da temperatura de metais
incandescentes em fundições.
Um dos pirómetros mais comuns é o de
absorção-emissão, que é utilizado para determinar a
temperatura de gases através da medição da radiação
emitida por uma fonte de referência, antes e depois da
radiação incidir sobre o gás (que absorve parte da
radiação). É através da análise das diferenças do
espectro do gás que se consegue determinar a sua
temperatura. Ambas as medições são feitas no mesmo
intervalo de comprimento de onda.
Outra aplicação típica do pirómetro é a
medição da temperatura de metais incandescentes.
Olhando pelo visor do pirómetro observa-se o metal,
ajustando-se depois manualmente a corrente elétrica
que percorre um filamento que está no interior do
pirómetro e aparece no visor. Quando a cor do
filamento é idêntica à do metal, pode-se ler a
temperatura numa escala disposta junto ao elemento de
ajuste da cor do filamento.
42
Física Moderna – Capítulo 4 - Radiação de corpo negro e Efeito Fotoelétrico
Prof. Dr. Cláudio S. Sartori
43
 Descoberto por acaso o sucessor das átomos, o que é justamente o "tamanho mágico" no
qual o cristais preferencialmente se formam. Assim,
lâmpadas incandescentes
Redação do Site Inovação Tecnológica
esses minúsculos pontos quânticos são fáceis de
25/10/2005
serem produzidos, ainda que tenham apenas metade
http://www.inovacaotecnologica.com.br/
do tamanho dos pontos quânticos normais.
Quando esses pontos quânticos foram
Pegue um LED que produza uma luz azul
intensa. Recubra-o com uma finíssima película de iluminados com um laser, ao invés da luz azul que os
cristais microscópicos, chamados pontos quânticos, e estudantes esperavam, eles se encantaram com o
você terá a próxima revolução tecnológica na branco vivo que iluminou a mesa onde faziam seu
iluminação, que poderá substituir virtualmente todas as experimento.
A seguir os estudantes dissolveram seus
atuais lâmpadas.
pontos
quânticos
em uma espécie de verniz para
Esse LED híbrido, descoberto por acaso pelo
madeira
e
"pintaram"
um LED. Embora isso seja o
estudante Michael Bowers, da Universidade
Vanderbilt, Estados Unidos, é capaz de emitir luz que se poderia chamar de uma típica uma idéia de
branca verdadeira, similar à emitida pelas lâmpadas estudante, eles estavam, na verdade, montando sua
descoberta sobre uma fonte própria de luz,
incandescentes, com uma leve tonalidade de amarelo.
Até agora os pontos quânticos têm recebido dispensando o laser. O resultado não é nenhum
atenção graças à sua capacidade de produzir dezenas primor de acabamento, mas demonstra claramente
de cores diferentes simplesmente variando-se o que a junção dos dois pode gerar uma nova fonte de
tamanho dos nanocristais individuais: uma capacidade luz branca que poderá revolucionar todo o setor de
particularmente adequada à marcação fluorescente de iluminação.
A descoberta foi descrita em um artigo
células em aplicações biomédicas.
Mas os cientistas agora descobriram uma publicado no exemplar de 18 de Outubro do Jornal da
nova forma para construir pontos quânticos capazes de Sociedade Americana de Química.
produzir espontaneamente luz branca de largo
espectro.
Até 1993 os LEDs só produziam luzes
vermelha, verde e amarela. Foi então que o
pesquisador japonês Isamu Akasaki descobriu como
fabricar LEDs que emitiam luz azul. Combinando
LEDs azuis com outros verdes e vermelhos - ou
adicionando-se fósforo amarelo aos LEDs azuis - os
fabricantes conseguiram criar luz branca, o que abriu
uma gama totalmente nova de aplicações para essas
fontes de luz, por natureza extremamente econômicas
e duráveis. Mas a luz emitda pelos "LEDs brancos" é
apenas ligeiramente branca, apresentando um forte
tom azulado.
Os pontos quânticos de luz branca, por outro
lado, produzem uma distribuição mais suave dos
comprimentos de onda do espectro visível, com uma
leve tonalidade amarela. Desta forma, a luz produzida
pelos pontos quânticos se parece mais com as luzes de
"espectro total" utilizadas para leitura, um tipo de
lâmpada disponível no mercado que produz uma luz
com um espectro mais próximo ao da luz do Sol do
que as lâmpadas incandescentes ou fluorescentes.
Além disso, os pontos quânticos, como
acontece também com os LEDs, têm a vantagem de
não emitir grandes quantidades de luz infravermelha,
como acontece com as lâmpadas incandescentes. Essa
radiação invisível produz grandes quantidades de calor
e é responsável pela baixa eficiência energética desse
tipo de lâmpada.
Bowers estava estudando com seu colega
James McBride, procurando entender como os pontos
quânticos crescem. Para isso eles estavam tentando
criar pontos quânticos cada vez menores. Foi então
que eles criaram um lote desses nanocristais de cádmio
e selênio. Esses elementos contêm 33 ou 34 pares de
43
Física Moderna – Capítulo 4 - Radiação de corpo negro e Efeito Fotoelétrico
Prof. Dr. Cláudio S. Sartori
44
44
Física Moderna – Capítulo 4 - Radiação de corpo negro e Efeito Fotoelétrico
Prof. Dr. Cláudio S. Sartori

Exemplos Resolvidos
1. Determine o fluxo de calor através da barra de cobre
de seção quadrada da figura.
Dado: condutividade térmica do cobre:  Cu 
388 J(s m°C)-1

dQ   A  T

dt
e
5. Para as radiações abaixo, dados os
intervalos extremos de comprimento de onda,
encontre os intervalos correspondentes em freqüência
(Hz) e energia (eV).
Espectro
visível
Visible
Cores
min    max fmin  f  f max Emin  E  Emax
(eV)
(nm)
(1014 Hz)
f 
45
4
dQ   A  T 388  4 10  100  0 



=77,6 cals
dt
e
0, 2
2. Tira-se de uma fornalha uma peça fundida
pesando 50 kgf, quando a temperatura era de 400°C,
sendo colocada num tanque contendo 400 kg de óleo a
30°C. A temperatura final é de 40°C e o calor
específico do óleo, 0,5 cal-g-1 (0C)-1. Qual o calor
específico da peça fundida? Desprezar a capacidade
calorífica do tanque e quaisquer perdas de calor.
Qo  Qp  0  mo  co  o  mp  c p   p  0
400  0,5   40  30   50  c p   40  400   0
c
1240
  nm 

Red –
Vermelho
622 -770
3,896 –
4,823
1,61 – 1,99
Orange –
Laranja
597 - 622
4,823 –
5,025
1,99 – 2,08
Yellow –
Amarelo
577 - 597
Green –
Verde
492 - 577
Blue – Azul
455 - 492
Violet –
Violeta
390 - 455
c  f  f 
f 
c

3 10
 f max  4,823 1014
9
622 10
f 
8
3 108
 f min  3,89611014
770 109
E  h f
h  6,62 1034  J  s 
c p  0,11 gcal0C
E  h
3. A evaporação do suor é um mecanismo
importante no controle da temperatura em animais de
sangue quente. Que massa de água deverá evaporar-se
da superfície de um corpo humano de 80 kg para
resfriá-lo 1°C? O calor específico do corpo humano é
aproximadamente l cal g -1 • (°C) -1 e o calor latente de
vaporização da água na temperatura do corpo (37°C) é
de 577 cal • g -1.
Quantidade de calor perdida pelo corpo
humano na variação de 10C:
E  eV  
c

E  6, 62 1034 
EJ  
3 108
1,986 10
  m

25
1eV=1,6 10-19J
1
1,986 1025
  nm  1, 6 1019 109
Q  mc  Q  80000 11  80000cal
1240
Q
80000
E  eV  
QL  mLv  m  L 
 138.65 g
  nm 
Lv
577
1240
4. Uma certa massa de gás tem o volume de
Emin  eV  
 Emin  eV   1, 61
2,5 L a 40°C na pressão de 1,5 atm. Se o volume do
770
gás for reduzido para 0.5L e aquecido a 70°C, qual sua
1240
nova pressão?
Emin  eV  
 Emax  eV   1,99
622
P1  V1 P2  V2
T2  V1
343  2,5
T1

T2
 P2 
T1 V2
P1 
313  0,5
E  eV  
1,5  8.22atm
45
Física Moderna – Capítulo 4 - Radiação de corpo negro e Efeito Fotoelétrico
Prof. Dr. Cláudio S. Sartori
 Exercícios
1. Condução através de uma geladeira de
isopor. Uma caixa de isopor usada para manter
bebidas frias em um piquenique possui área total
(incluindo a tampa) igual a 0.80m2 e a espessura da
parede é de 2.0 cm. Ela está cheia de água, gelo e latas
de Omni-Cola a 00C. Qual é a taxa de fluxo de calor
para o interior da caixa se a temperatura da parede
externa for igual a 300C? Qual a quantidade de gelo
que se liquefaz durante um dia?
Dado: isopor=0.010 W/(m.K)
46
4. Radiação do corpo humano. Sabe-se que a
área total do corpo humano é igual a 1.20m2 e que a
temperatura da superfície é 300C = 303K. Calcule a
taxa total de transferência de calor do corpo por
radiação. Se o meio ambiente está a uma temperatura
de 200C, qual é a taxa resultante do calor perdido
pelo corpo por radiação? A emissividade e do corpo é
próxima da unidade, independentemente da cor da
pele.
Dados: Lei de Stefan-Boltzmann:
H  A  e   T 4
H  A  e    T 4  Ts4


Constante de Stefan-Boltzmann:
  5.67 108
W
m K4
2
5. Uma placa quadrada de aço, com lado igual a
10 cm, é aquecida em uma forja de ferreiro até uma
temperatura de 8000C. Sabendo que a emissividade é
2. Uma barra de aço de 10.0 cm de comprimento é igual a 0.60, qual é a taxa total de energia transmitida
soldada pela extremidade com uma barra de cobre de por radiação?
20.0 cm de comprimento. As duas barras são
6. Um chip com embalagem de cerâmica de 40
perfeitamente isoladas em suas partes laterais. A seção
pinos
possui rtérm = 40 K/W. Se a temperatura
reta das duas barras é um quadrado de lado 2.0 cm. A
0
máxima
que o circuito pode tolerar com segurança
extremidade livre da barra de aço é mantida a 100 C
não
pode
superar 1200C, qual é o mais elevado nível
colocando-a em contato com vapor d’água obtido por
ebulição, e a extremidade livre da barra de cobre é de potência que o circuito pode tolerar com
para uma temperatura ambiente igual a
mantida a 00C colocando-a em contato com o gelo. segurança
0
75
C?
Calcule a temperatura na junção entre as duas barras e
a taxa total da transferência de calor.
dQ   A  T

dt
e
7. Tira-se de uma fornalha uma peça fundida
pesando 50 kgf, quando a temperatura era de 400°C,
sendo colocada num tanque contendo 400 kg de óleo
e
a 30°C. A temperatura final é de 40°C e o calor
R
específico do óleo, 0,5 cal-g-1 (0C)-1. Qual o calor
A
3. No exemplo anterior, suponha que as barras específico da peça fundida? Desprezar a capacidade
estejam separadas. Uma extremidade é mantida a calorífica do tanque e quaisquer perdas de calor.
1000C e a outra extremidade é mantida a 00C. Qual a
taxa total de transferência de calor nessas duas barras? Qo  Qp  0  mo  co  o  mp  c p   p  0

400  0,5   40  30   50  c p   40  400   0
c p  0,11 gcal0C
8. A evaporação do suor é um mecanismo
importante no controle da temperatura em animais de
sangue quente. Que massa de água deverá evaporarse da superfície de um corpo humano de 80 kg para
resfriá-lo 1°C? O calor específico do corpo humano é
46
Física Moderna – Capítulo 4 - Radiação de corpo negro e Efeito Fotoelétrico
Prof. Dr. Cláudio S. Sartori
aproximadamente l cal g -1 • (°C) -1 e o calor latente de
vaporização da água na temperatura do corpo (37°C) é
de 577 cal • g -1.
Quantidade de calor perdida pelo corpo
humano na variação de 10C:
Q  mc  Q  80000 11  80000cal
Q
80000
QL  mLv  m  L 
 138.65 g
Lv
577
47
E  eV  
Emin  eV  
1240
  nm 
1240
 Emin  eV   1, 61
770
10. Área do filamento de uma lâmpada de
tungstênio. A temperatura de operação do filamento
de tungstênio de uma lâmpada incandescente é igual
9. Para as radiações abaixo, dados os
a 2450K e sua emissividade é igual a 0.35. Calcule a
intervalos extremos de comprimento de onda, encontre
área da superfície do filamento de uma lâmpada de
os intervalos correspondentes em freqüência (Hz) e
150 W supondo que toda a energia elétrica
energia (eV).
consumida pela lâmpada seja convertida em ondas
eletromagnéticas pelo filamento. (Somente uma
Espectro Cores     
f

f

f
E

E

E
min
max
min
max
min
max fração do espectro irradiado corresponde à luz
visível
visível.)
Visible
(nm)
(1014 Hz)
(eV)
1240
11. Raios de estrelas. A superfície quente e
c
E  eV  
f 
  nm 
brilhante
de uma estrela emite energia sob a forma de

radiação eletromagnética. É uma boa aproximação
Red –
622 -770
3,896 –
1,61 –
considerar e = 1 para estas superfícies. Calcule os
Vermelh
4,823
1,99
raios das seguintes estrelas (supondo que elas sejam
o
esféricas):
Orange – 597 - 622
4,823 –
1,99 –
(a) Rigel, a estrela brilhante azul da
Laranja
5,025
2,08
constelação Órion, que irradia energia com uma taxa
Yellow – 577 - 597
de 2.7.1032W e a temperatura na superfície é igual a
Amarelo
11000K.
Green –
492 - 577
(b) Procyon B (somente visível usando um
Verde
telescópio), que irradia energia com uma taxa de
Blue –
455 - 492
2.1.1023W e a temperatura na sua superfície é igual a
Azul
10000K.
Violet –
390 - 455
(c) Compare suas respostas com o raio da
Violeta
Terra, o raio do Sol e com a distância entre a Terra e
o Sol. (Rigel é um exemplo de uma estrela
supergigante e Procyon B é uma estrela anã branca.
c
c  f  f 
f 

3 10
 f max  4,823 1014
622 109
f 
8
12. Determine o comprimento da barra
indicado para que o fluxo de calor seja de 250W.
3 108
 f min  3,89611014
770 109
E  h f
h  6,62 1034  J  s 
E  h
c

E  6, 62 1034 
3 108

Dados: condutividade térmica:
cobre:  Cu  385,0 J(s m°C)-1
aço:  Aço  50,2 J(s m°C)-1
1,986 1025
  m
1eV=1,6 10-19J
1
1,986 1025
E  eV  
  nm  1, 6 1019 109
EJ  
47
Física Moderna – Capítulo 4 - Radiação de corpo negro e Efeito Fotoelétrico
Prof. Dr. Cláudio S. Sartori
13. A Lei do deslocamento de Wien é
obtida, impondo-se
T
0

Para:
 T  d 
8hc

1
5
e
hc
 k T
d
1
Utilizando a Lei do deslocamento de Wien:
max 
48
2.898
 mm  K 
T
Ache a que temperatura corresponde ao máximo
comprimento de max = 305 nm.
16. A temperatura superficial do Sol é cerca
(b) Aplicando a Lei de Stefan-Boltzman:
de 6000K.
4
H  A  e   T
(a) Se admitirmos que o Sol irradia como
: constante de Stefan-Boltzmann.
um
corpo
negro, em que comprimento de onda max
2
  5.6699 108 WKm4
se localizará o máximo da distribuição espectral?
Encontre a potência dissipada nessa temperatura,
(b) Calcular max para um corpo negro a
assumindo área 20 cm2 e emissividade e = 1;
temperatura ambiente, cerca de 300 K.
14. Duas barras metálicas, cada qual com 5
cm de comprimento e seção reta retangular de 2 cm
por 3 cm, estão montadas entre duas paredes, uma
mantida a 100 0C e outra a 0 0C. Uma barra é de
chumbo (Pb) e a outra é de prata (Ag). Calcular:
(a) A corrente térmica através das barras.
(b) a temperatura da superfície de contato das
duas.
Dado: Condutividades térmicas:
Pb = 353 W/(m.K)
Ag = 429 W/(m.K)
17. Calcular a perda de energia líquida de
uma pessoa nua numa sala a 200C, admitindo que
irradie como um corpo negro de área superficial igual
a 1.4 m2, na temperatura de 33 0C. A temperatura
superficial do corpo humano é ligeiramente mais
baixa que a temperatura interna de 370C, em virtude
da resistência térmica da pele.
18. Na prática de construção civil, nos países
de língua inglesa, especialmente nos Estados Unidos,
é costume utilizar o fator R, simbolizado por Rf, que é
a resistência térmica por pé quadrado do material.
Assim, o fator R é igual ao quociente entre a
espessura do material e a condutividade térmica:
Rf 
e

 R A
A tabela ilustra os fatores de R para alguns
materiais de construção.
Tabela 1 – Fatores R para alguns materiais
de construção.
e
15. As duas barras do exemplo anterior são
montadas como ilustra a figura a seguir. Calcular:
(a) A corrente térmica em cada barra
metálica.
(b) A corrente térmica total.
(c) A resistência térmica equivalente desta
montagem.
Rf
Material
(in)
Chapas divisórias
Gesso ou estuque
Compensado
(pinho)
Painéis de madeira
Carpetes
Isolamento de teto
0.375
(h.ft2.F/Btu)
0.32
0.5
0.62
0.75
1.0
1.0
0.93
2.08
2.8
Manta asfáltica
1.0
0.15
Chapas de madeira
asfáltica
1.0
0.44
48
Física Moderna – Capítulo 4 - Radiação de corpo negro e Efeito Fotoelétrico
Prof. Dr. Cláudio S. Sartori
Um telhado de 60 ft por 20 ft é feito de chapa
22. – O espectro típico de uma lâmpada
de pinho, de 1 in, cobertas por chapas de madeira fluorescente está indicado abaixo:
asfáltica.
(a) Desprezando a superposição das chapas de
madeira, qual a taxa de condução de calor através do
telhado, quando a temperatura no interior da
edificação for de 70 0F e no exterior 40 F ?
(b) Calcular a taxa de condução de calor se à
cobertura anterior forem superpostas 2 in de
isolamento especial para telhados.
19. A equação:
49
(a) Utilizando a Lei do deslocamento de
F
 Y    T
A
Fornece a tensão necessária para manter a
temperatura da barra constante à medida que a
temperatura varia. Mostre que se o comprimento
pudesse variar de ΔL quando sua temperatura varia de
ΔT, a tensão seria dada por:
 L

F  A Y  
   T 
 L0

Onde:
F: tensão na barra.
L0: comprimento original da barra.
Y: Módulo de Young.
Α: coeficiente de dilatação linear.
20. Uma placa quadrada de aço de 10 cm de
lado é aquecida em uma forja de ferreiro até 100 0C. Se
sua emissividade é e = 0.60, qual será a taxa total de
energia emitida por radiação ?
21 - Determine:
(a) As resistências térmicas do cobre, do aço
e a equivalente.
(b) O fluxo de calor através da barra de cobre
de seção quadrada da figura. A temperatura na
interface.
Dados: condutividade térmica:
cobre:  Cu  385,0 J(s m°C)-1
aço:  Aço  50,2 J(s m°C)-1
dQ   A  T


dt
e
R
e
A
Wien:
max 
2.898
 mm  K 
T
Ache a que temperatura corresponde ao máximo
comprimento de onda dessas lâmpadas. Observe que
o pico em comprimento de onda ocorre para essas
lâmpadas em torno de max = 305 nm.
(b) Aplicando a Lei de Stefan-Boltzman:
H  A  e   T 4
: constante de Stefan-Boltzmann.
  5.6699 108 WKm
2
4
Encontre a potência dissipada nessa temperatura,
assumindo área 20 cm2 e emissividade e = 1;
23. – As lâmpadas UV fluorescentes são
usualmente categorizadas como lâmpadas UVA,
UVB ou UVC, dependendo da região em que maior
parte de sua irradiação se situa. O espectro UV está
dividido dentro de três regiões:
 Região UVA, 315 a 400 nanômetros;
 Região UVB, 280 a 315 nanômetros;
 Região UVC, abaixo de 280 nanômetros.
Complete a relação da tabela.
Região

0
(A)
UVA
> 109
UVB
109 106
UVC
106 7000
Visível
f
(Hz)
E
(eV)
< 3 x 109
< 10-5
10-5 - 0.01
3 x 1012
- 4.3 x
1014
4.3 x
1014 7.5 x
1014
2-3
Dados: f  c ; E  h  f h  6,62 1034  J  s  ;

c= 3.108m/s;
1240
E  eV  
  nm 
49
Física Moderna – Capítulo 4 - Radiação de corpo negro e Efeito Fotoelétrico
Prof. Dr. Cláudio S. Sartori
24. – Se colocarmos as barras indicadas numa
ligação em paralelo encontre a resistência térmica
equivalente e o fluxo total de calor.

Solução:
k  A  T
kA
 dr 
dT
e

dr 4 k
k  4  r 2
dT
dr 
dT  2 
r


R2
T
dr 4 k 2

R r 2  T dT
1
1

50
Dados: condutividade térmica:
cobre:  Cu  385,0 J(s m°C)-1
aço:  Aço  50,2 J(s m°C)-1

dQ   A  T
e
R

A
dt
e
25. – Explique o mecanismo das brisas
oceânicas.
r R

1 2 4 k T T2

T
r r  R1
 T T1
1
1 4 k
 
T2  T1 
R2
R1

1 1 4 k
  
T2  T1 
R2 R1

R2  R1 4 k

T2  T1 
R1  R2

4 k  R1  R2

T2  T1 
R2  R1

28. O raio interno a de uma casca cilíndrica
está mantido a uma temperatura Ta enquanto seu raio
externo b está a uma temperatura Tb. A casca
cilíndrica possui uma condutividade térmica
uniforme k. Mostre que o fluxo sobre a casca
26. – Determine o comprimento da barra cilíndrica é dada por:
indicado para que o fluxo de calor seja de 250W.
 T T 
  2  L  k  a b 
 ln  b a  
27. Uma camada esférica de condutividade
térmica k tem um raio interno r1 e um raio externo r2.
A camada interna está a uma temperatura T1 e
a externa a uma temperatura T2. Mostre que a corrente
térmica é dada por:

4 k  r1  r2 T2  T1 
r2  r1
50
Física Moderna – Capítulo 4 - Radiação de corpo negro e Efeito Fotoelétrico
Prof. Dr. Cláudio S. Sartori


51
Solução:
k  A  T
kA
 dr 
dT
e

k  2  r  L
dr 
dT

dr 2  k  L

dT
r

T
b
dr 2  k  L b
a r   T dT
a
2  k  L T Tb
T T T
a

2  k  L
ln b  ln a 
Tb  Ta 

2  k  L
ln  b a  
Tb  Ta 

r b
ln r r a 
 T T 
  2  L  k  b a 
 ln  b a  
Fluxo de dentro para fora.
Fluxo de fora para dentro:
 T T 
  2  L  k  b a 
 ln  b a  
 T T 
  2  L  k  a b 
 ln  b a  
29. A seção de passageiros de uma vião a jato
tem a forma de uma tubulação cilíndrica com 35m de
comprimento e raio interno 2.50m. Sabe-se que a
espessura do tubo que compõe o avião é cerca de 6 cm
e tem uma condutividade térmica dada por 4.10 5
cal/(s.cm°C). A temperatura deve ser mantida dentro
em cerca de 25°C e fora do avião na altitude de
cruzeiro é cerca de -35°C. Que potência deve ser feita
para que se mantenha essa diferença de temperatura?

Solução:
51
Física Moderna – Capítulo 4 - Radiação de corpo negro e Efeito Fotoelétrico
Prof. Dr. Cláudio S. Sartori
O Efeito Fotoelétrico
Um dos fenômenos fundamentais para a
comprovação da teoria quântica é o efeito fotoelétrico
e o efeito Compton. Ambos envolvem a interação
entre a radiação eletromagnética e os elétrons
presentes na matéria.
A condução elétrica por meio de descarga gasosa
era um fenômeno comum de investigação na segunda
metade do século XIX. Um arranjo experimental típico
é mostrado na figura 1 abaixo.
52
Figura 1 – Tubo de raios catódicos. Aparato
experimental na investigação de corrente elétrica
gerada por descarga gasosa na segunda metade do
século XIX.
Quando a pressão no tubo é diminuída da
ordem de 0.01 mmHg ou menos, o gás para de
brilhar. Porém, o amperímetro ainda indica a
passagem de corrente elétrica. Outro fenômeno
observado é a presença de um brilho em S, na
superfície interior do tubo. O tamanho do spot S
(brilho) depende do tamanho e da localização dos
diafragmas D e D’. A posição da luminosidade do
spot pode ser influenciada pela presença de um forte
campo elétrico e magnético colocados numa região à
direita de D’, comprovando-se que as partículas que
atravessam a região estão eletricamente carregadas;
um sensor de posição indica que a carga das
partículas é negativa. Estas partículas, que
atravessam como um raio do cathode eram chamadas
de partículas de raios catódicos, ou simplesmente
raios catódicos e o aparato indicado na figura 1 é
chamado de tubo de raios catódicos.
Hoje sabemos que as partículas são emitidas
do cathodo como resultado do bombardeamento de
íons positivos do gás contidos no tubo.
Figura 2 – Aparato utilizado por J.J.
Thomson para encontrar a razão e/m do elétron.
Em 1897, Thomson mediu a razão entre a
carga e e a massa m do elétron com o aparato descrito
acima. As partículas de raios catódicos, de carga
negativa, atravessavam do cathodo C ao anodo A, o
qual era perfurado de um pequeno buraco, atuando
como o primeiro de dois diafragmas, D e D’, As
partículas passam por duas placas metálicas paralelas
de comprimento l e distanciadas de d. Finalmente
elas se chocam no final do tubo, produzindo uma
luminosidade S1.
A aplicação de uma voltagem V entre as
placas cria uma força que atua nas partículas numa
direção perpendicular à velocidade v, dada por:
F
Uma alta voltagem era aplicada entre o
eletrodo negativo (cathode) e o eletrodo positivo
(anode), que causava um fluxo de corrente no tubo de
vidro, o qual era preenchido por certos gases a uma
baixa pressão. A corrente provoca camadas brilhantes
adjacentes de gás envoltas por camadas mais escuras,
cuja natureza depende da pressão do gás e do valor da
voltagem aplicada. Um conjunto complicado de
fenômenos é responsável pela condução elétrica e
brilho das camadas gasosas, cuja luz é emitida pelas
moléculas do gás causada pela sua ionização.
Essa força
transversal dada por:
e V
d
produz
a
uma
aceleração
e V
md
Ela atua durante um tempo l/v durante o qual
as partículas estão atravessando a região entre as
placas. Após sair da região entre as placas, a deflexão
sofrida pelas partículas na direção transversal será:
1
2
  a t2
52
Física Moderna – Capítulo 4 - Radiação de corpo negro e Efeito Fotoelétrico
Prof. Dr. Cláudio S. Sartori
1 eV

2md
A
deflexão
l
 
v
2
S1S2 do spot luminoso é
amplificada, para pequenas deflexões, por um fator
2L/l, onde L é a distância do centro das placas até o
fim do tubo.
2
1 e V 2L  l 
S1S2 
 
2 m d l v
Se a deflexão S1S 2 é medida e a velocidade v
53
das partículas é conhecida, então o valor e/m será
descoberto.
Foi em 1886 e 1887 que Heinrich Hertz realizou
as experiências que pela primeira vez confirmaram a
existência de ondas eletromagnéticas e a teoria de
Maxwell sobre a propagação da luz. É um desses fatos
paradoxais e fascinantes na história da ciência que
Hertz tenha notado, no decorrer de suas experiências,
o efeito que Einstein mais tarde usou para contradizer
outros aspectos da teoria eletromagnética clássica.
Hertz descobriu que uma descarga elétrica entre dois
eletrodos ocorre mais facilmente quando se faz incidir
sobre um deles luz ultravioleta.
Lenard, seguindo alguns experimentos de
Hallwachs, mostrou logo em seguida que a luz
ultravioleta facilita a descarga ao fazer com que
elétrons sejam emitidos da superfície do cátodo. A
emissão de elétrons de uma superfície, devida à
incidência de luz sobre essa superfície, e' chamada
efeito fotoelétrico.
A figura 3 mostra um aparelho usado para estudar
o efeito fotoelétrico.
Figura 3 –
Um invólucro de vidro encerra o aparelho
em um ambiente no qual se faz vácuo. Luz
monocromática, incidente através de uma janela de
quartzo, cai sobre a placa de metal e libera elétrons
chamados fotoelétrons. Os elétrons podem ser
detectados sob a forma de uma corrente se forem
atraídos para o coletor metálico através de uma
diferença de potêncial V estabelecida. Um
amperímetro mede essa corrente fotoelétrica.
A curva da figura 4 e um gráfico da corrente da
fotoelétnca, em um aparelho como o figura 3 em
função da diferença de poteucial. Se V é muito
grande, a corrente foloelétrica atinge um certo valor
limite (ou de saturação) no qual todos os fotoelétrons
em emitidos por A são coletados por B.
Se o sinal de V é invertido, a corrente
fotoelétrica não cai imediatamente a zero, o que
sugere que os elétrons são emitidos de A com alguma
energia cinética. Alguns alcançarãocoletor B apesar
do campo elétrico opr-se ao seu movimento.
Entretanto, se essa diferença de potencial tornar-se
suficientemente grande, um valor V0 chamado
53
Física Moderna – Capítulo 4 - Radiação de corpo negro e Efeito Fotoelétrico
Prof. Dr. Cláudio S. Sartori
54
potencial limite ou de corteé atingido, e a corrente
Há três aspectos do efeito fotoelétrico que
fotoelétrica cai a zero. Essa diferença de potencial V0, não podem ser explicados em termos da teoria
multiplicada pela carga do elétron –e, mede a energia ondulatória clássica da luz:
cinética máxima Kmax do mais rápido fotoelétron
1. A teoria ondulatória requer que a
emitido. Isto é:
amplitude do campo elétrico oscilante E da onda
luminosa cresça se a intensidade da luz for
Kmax  e V0
aumentada. Já que a força aplicado ao elétron é e.E,
Experimentalmente nota-se que a quantidade
isso sugere que a energia cinética dos fotoelétrons
Kmax é independente da intensidade da luz incidente,
deveria crescer também ao se aumentar a intensidade
como é mostrado na figura 4 (a), na qual a intensidade
do feixe luminoso.Entretanto, como vimos na figura
da luz foi reduzida à metade para se obter a curva I. Já
anterior, Kmax = e.V0, independe da intensidade da
para freqüências diferentes, observa-se o mostrado na
luz. Isto foi testado para variações de intensidade da
figura 4 (b).
ordem de 107.
2. De acordo dom a teoria ondulatória, o
Figura 4 (a)
efeito fotoelétrico deveria ocorrer para qualquer
freqüência da luz, desde que fosse intensa o bastante
para dar a energia necessária à ejeção dos elétrons.
Entretanto, a figura 5 mostra que existe, para cada
superfície, um limiar de freqüências 0
característico. Para freqüências menores que 0, o
efeito fotoelétrico não ocorre, qualquer que seja a
intensidade da iluminação.
Figura 5 –
Figura 4 (b)
3. Se a energia requerida por um fotoelétron
é absorvida da onda incidente sobre a placa metálica,
a área de alvo efetiva para um elétron no metal é
limitada, e provavelmente não é muito maior que a de
um círculo de raio aproximadamente igual ao raio
atômico. Na teoria clássica, a energia luminosa está
uniformemente distribuída sobre a frente de onda.
Portanto, se a luz é suficientemente fraca, deveria
haver um intervalo de tempo mensurável, que vamos
estimar, entre o instante que a luz começa a incidir na
superfície e o instante da ejeção do fotoelétron.
Durante este intervalo, o elétron deveria estar
absorvendo energia do feixe, até que estivesse
acumulado o bastante para escapar. No entanto,
nenhum retardamento detectável foi jamais medido.
Esta
discordância
é
particularmente
marcante quando a substância fotoelétrica for um
gás; nessas circunstâncias, mecanismos de absorção
coletivos podem ser ignorados e a energia do
fotoelétron emitido deve certamente ter sido extraída
do feixe luminoso por um único átomo ou molécula.
54
Física Moderna – Capítulo 4 - Radiação de corpo negro e Efeito Fotoelétrico
Prof. Dr. Cláudio S. Sartori
55
 Exemplo 1 - Uma placa de potássio
é colocada a l m de uma fonte luminosa pouco intensa,
cuja potência é l W ; l joule/s. Suponha que um
fotoelétron ejetado possa ter coletado sua energia em
uma arca circular da placa, cujo raio r é, digamos, um
raio atómico: r = 1.10-10m. A energia necessária para
remover um elétron da superfície de potássio é cerca
de 2.1 eV = 3.4.10-19 J (Joule).
(Um elétron-volt: l eV = l.6.10-19 J (joule) é a
energia ganha por um elétron, cuja carga é –e = -l.6.1019
C (Coulomb), ao passar através de uma diferença de
potencial de l V (Volt).) Quanto tempo levaria o
elétron para absorver essa quantidade de energia da
fonte luminosa ?

Solução:
Suponha que a energia está uniformemente
distribuída sobre a frente de onda. A área de alvo é:
 r2 = .10-20m2 . A área de uma esfera com r
de raio é 4r2 com o raio centrada na fonte. Se a fonte
irradia uniformemente em todas as direçõcs (isto é, se
a energia está uniformemente distribuída sobre frentes
de onda esféricas que se afastam da tome, de acordo
com a teoria clássica) a taxa por segundo a qual a
energia incide sobre o alvo é:
 1020
R  1J s 
 R  2.5 1021 J s
2
4 1
Supondo que toda essa potência é absorvida,
podemos calcular o tempo necessário para um eletron
para adquirir energia suficiente para escapar; e
encontramos:
t
3.4 1019
 t  1.4 102 s
21
2.5 10
t  2 min
É claro que poderíamos diminuir o tempo
calculado supondo uma área efetiva maior. A hipótese
mais favorável, que a energia é transmitida por um
processo de ressonância da onda luminosa para o
elétron, leva a uma área de alvo igual a 2 onde  é o
comprimento de onda da luz, mas ainda obteríamos
um retardamento finito que estaria bem dentro de
nossas possibilidades de medida experimental. (Para a
luz ultravioleta de t  10-2s, por exemplo, entretanto,
não foi detectado nenhum retardamento em quaisquer
circunstâncias, as primeiras experiências fixando um
limite superior de 10-9s para qualquer retardamento
possível!

A TEORIA QUÂNTICA DE EINSTEIN
SOBRE O EFEITO FOTOELÉTRICO
Em 1905 Einsten colocou em questão a teoria
clássica da luz, propôs uma nova teoria, e citou o
efeito fotoelétrico como uma aplicação que poderia
testar qual teoria estava correta. Isto aconteceu
vários anos antes do trabalho de Millikan, mas
Einstein foi influenciado pela experiência de Lenard.
Como já mencionamos, Planck originalmente
restringiu seu conceito de quantização de energia aos
elétrons nas paredes de um corpo negro. Planck
acreditava que a energia eletromagnética, uma vez
irradiada, se espalhava pelo espaço da mesma forma
que ondas de água se espalham na água. Em vez
disso, Einstein propôs que a energia radiante está
quantizada em pacotes concentrados, que mais tarde
vieram a ser chamados fótons.
Einstein argumentou que as experiências óticas
bem conhecidas de interferência e difração da
radiação eletromagnética haviam sido feitas apenas
em situações que envolviam um número muito
grande de fótons. Estas experiências fornecem
resultados que são médias do comportamento dos
fótons individuais. A presença dos fótons nessas
experiências não é mais aparente do que a presença
de gotas d'água isoladas em um jato de água de uma
mangueira de jardim, se o número de gotas é muito
grande. Evidentemente, as experiências de
interferência e difração mostram definitivamente que
os fotons não vão de onde são emitidos até onde são
absorvidos da mesma maneira simples que partículas
clássicas, como as gotas d'água, o fazem. Eles se
propagam como ondas clássicas, no sentido que
cálculos baseados neste tipo de propagação clássico
(em particular, na maneira que dua.s ondas se
superpõem.
retorçando-se
ou
anulando-se
dependendo de sua fase relativa) explicam
corretamente em média como os fotons viajam.
Einstein não concentrou sua atenção na teoria
ondulatória familiar com que a luz se propaga, mas
sim na meneira corpuscular com que ela é emitida e
absorvida. Ele argumentou que a exigência de Planck
de que a energia das ondas eletromagneticas de
frequência  irradiadas por uma fonte (por exemplo,
uma fonte de luz ultravioleta numa experiência
fotoelétrica) fosse apenas 0, ou h., ou 2 h.,..., n h..
implicava que no processo de ida de um estado de
energia n h.. para um estado de eneigia (n+1) h..;
a fonte emitia um pulso discreto de radiação
eletromagnetica com energia:
E  h 
Supôs também que no processo fotoelétrico um
fóton é completamente absorvido por um elétron no
fotocatodo.
Quando um eletron é emitido da superfície do
metal, sua energia cinética é:
K  h   w
onde h., é a energia do fóton incidente absorvido e
w é o trabalho necessário para remover o elétron do
55
Física Moderna – Capítulo 4 - Radiação de corpo negro e Efeito Fotoelétrico
Prof. Dr. Cláudio S. Sartori
metal. Esse trabalho é necessário para superar os
campos atrativos dos átomos na superfície e as perdas
de energia cinética devidas ás colisões internas do
elétron. Alguns elétrons estão mais fortemente ligados
do que outros, alguns perdem energia por colisões em
sua trajetória. No caso da ligação mais fraca e
nenhuma perda interna, o fotoelétron vai emergir com
a energia cinética máxima Kmax. Portanto:
Kmax  h   w0
56
onde w0 é uma energia característica do metal chamada
função trabalho; é a energia mínima necessária para
um eletron atravessar a superfície do metal e escapar
ás forças atrativas que normalmente ligam o eletron ao
metal.
Vejamos agora como a hipótese de Einstein
resolve as três objeções levantadas contra a
interpretação ondulatória do efeito fotoelétrico.
Quanto á objeção 1 (o fato de que A'
não
depende da intensidade da iluminação), a teoria do
fóton concorda integralmente com a experiência.
Dobrar a intensidade da luz meramente dobra o
número de fótons e portanto duplica a corrente
fotoelétrica: isto não muda a energia h. de cada fóton
ou a natureza do processo fotoelétrico descrita.
A objeção 2 (a existência de um limiar de
frequências) é removida imediatamente por
Kmax  h   w0 . Se Kmax é igual a zero, temos:
h   w0
que significa que um fóton de frequência
0
tem exatamente a energia necessária para ejetar os
fotoelétrons, e nenhum excesso que possa aparecer
como energia cinética. Se a frequência for menor que
 0 , os fótons, não importando quantos eles sejam
(isto é, quão intensa seja a iluminação), não terão
individualmente a energia necessária para ejetar
fotoelétrons.
A objeção 3 (a ausência de retardamento) é
eliminada pela hipótese do fóton, pois a energia
necessária é fornecida em pacotes concentrados. Ela
não se espalha uniformemente sobre uma área
extensa, como supusemos no exemplo 1, suposição
esta baseada na hipótese de que a teoria ondulatória
clássica seja verdadeira. Se houver luz incidindo
sobre o cátodo, haverá pelo menos um fóton que o
atinge; este fóton será imediatamente absorvido por
algum átomo, causando a imediata emissão de um
fotoelétron.
Vamos reescrever a equação fotoelétrica de
Einstein substituindo e  V0 por K max :
V0 
h  w0

e
e
2.20V  0.65V
 3.9 1015V  s
10 10 14 6 1014
Podemos determinar h multiplicando esta
razão pela carga eletrônica e. Portanto:
h  3.9 1015 1.6 1019  h  6.2 1034 J  s
De uma análise muito mais cuidadosa desse
e de outros dados, inclusive dados obtidos com
superfícies de Lítio, Millikan achou o valor h =
6,57.10-34
j-s,
com
uma
precisão
de
aproximadamente 0,5%. Esta medida estava bem
próxima do valor de h deduzido da fórmula da
radiação de Planck. A concordância numérica das
duas determinações de h usando fenômenos e
teorias completamente diferentes, é notável.
Um valor moderno de h deduzido de
diversas experiências, é:
h  6.6262 1034 J  s
Citando Millikan: "O efeito fotoelétrico . . .
fornece uma prova independente da fornecida pela
radiação do corpo negro, da exatidao da hipótese
fundamental da teoria quântica, ou seja, a hipótese
da emissão descontinua ou explosiva da energia que
e absorvida das ondas pêlos constituintes
eletrônicos dos átomos. Ele materializa, por assim
dizer, a quantidade h descoberta por Planck em seu
estudo da radiação de corpo negro e, como nenhum
outro fenômeno, nos faz acreditar que o conceito
físico básico que esta por trás do trabalho de Planck
corresponde á realidade".

Exemplo 2 - Obter a função
trabalho para o sódio da figura.
4.39.1014
A interseçâo da linha reta na figura 2-3 com
o eixo horizontal é o limiar de frequências:
v0 = 4.39.1014Hz. Substituindo em:
h   w0  w0  6.63 1034  4.39 1014
w0  2.92 1019 J
1eV
w0  2.92 1019 J
1.6 1019 J
w0  1.82eV
O mesmo valor é obtido na figura para o módulo
Portanto a teoria de Einstein prevê uma
relação linear entre o potencial limite V0 e a da interseção da reta com o eixo vertical. Entretanto,
frequência v, em completa concordância com experiências modernas dão o valor mais alto 2,27 eV.
Para a maioria dos metais o valor da função
resultados experimentais. A inclinação da curva
trabalho e da ordem de poucos elétrons-volt é igual
experimental da figura deve ser h/e, portanto:
56
Física Moderna – Capítulo 4 - Radiação de corpo negro e Efeito Fotoelétrico
Prof. Dr. Cláudio S. Sartori
ao valor da função trabalho para a emissão
termoiônica desses metais.
*
 Exemplo 3 - A que taxa por unidade de área
os fótons atingem a placa metálica do exemplo 1?
Suponha que a luz seja monocromática, de
comprimento de onda  = 5890 Å (luz amarela)
 Solução:
A taxa por unidade de área segundo a qual a
energia incide sobre uma placa metálica a l m de
distância de uma fonte luminosa de l W é:
R
57
1 Js
J
 R  8 102 2
2
4 1
m s
eV
R  5 1017 2
m s
Cada fóton tem uma energia de:
E  h   E  h 
c

3 108
5.89 107
E  3.4 1019 J
E  2.1eV
E  6.63 1034 
Portanto a taxa R segundo a qual os fótons
atingem uma unidade de área da placa é:
eV 1 fóton

m2  s 2.1eV
fóton2
R  2.4 1017 2
m s
R  5 1017
elogiando Einstein, na qual escreveram: "Em resumo,
podemos dizer que dificilmente haverá um grande
problema, dos quais a física moderna e tão rica, ao
qual Einstein não tenha dado uma importante
contribuição. Que ele tenha algumas vezes errado o
alvo em suas especulações, como por exemplo em
sua hipótese dos quanta de luz (fótons), não pode ser
realmente colocado contra ele, poi é impossível
introduzir ideias fundamentalmente novas, mesmo
nas ciências mais exalas, sem ocasionalmente correr
um risco".
Hoje a hipótese do fóton é usada em todo o
espectro eletromagnético, não apenas na região
visível. Pode-se dizer, por exemplo, que uma
cavidade de microondas contém fótons. Com  = 10
cm, um comprimento de onda típico de microondas,
pode-se calcular, da mesma forma que anteriormente,
que a energia do fóton é 1,20 x 10 s eV. Esta energia
é muito pequena para ejetar fotoelétrons de
superfícies metálicas. Para raios X, ou para raios 
tais como os que são emitidos por núcleos
radioativos, a energia do fóton pode ser de 106 eV, ou
mais. Estes fótons podem extrair de átomos pesados
elétrons fortemente ligados por energias da ordem de
10-5 eV. Os fótons na região visível do espectro
eletromagnético não tem energia suficiente para fazer
isto, e os fotoelétrons que eles ejetam são os
chamados elétrons de condução, que estão ligados ao
metal por energias de alguns elétrons-volt.
Observe que os fótons são absorvidos no
processo fotoelétrico. Isto requer que os elétrons
estejam ligados a átomos, ou sólidos, pois um elétron
completamente livre não pode absorver um fóton e
conservar simultaneamente a energia e o momento
relativísticos totais. Devemos ter um elétron ligado
para que as forças de ligação transmitam momento
para o átomo ou sólido.
O efeito fotoelétrico pode ocorrer porque a
energia do fóton é exatamente igual a funçao trabalho
de 2,1 eV para a superfície de potássio. Note que se o
comprimento de onda for ligeiramente aumentado (isto
é, se v for ligeiramente diminuída) o efeito fotoelétrico
deixará de ocorrer, não importando quão grande seja a
taxa R.
Este exemplo sugere que a intensidade I da luz
pode ser interpretada como o produto de N, o número
de fótons por unidade de área por unidade de tempo,
por hv a energia de um único fóton. Vemos aqui que
mesmo a intensidades relativamente pequenas (  10-1
W/m2 ) o número N é extremamente grande (  1017
fótons/(m2.s)) de modo que a energia de um único
fóton e muito pequena. Isto explica a extrema
granulosidade da radiação, e indica por que
normalmente e tão difícil detectá-la. Isto é análogo a
tentar detectar a estrutura atômica de uma porção de
matéria que, para a maioria dos fins, pode ser
considerada contínua, sua granulosidade sendo
revelada apenas em circunstancias especiais.
Em 1921 Einstein recebeu o Prêmio Nobel por ter
previsto teoricamente a lei do efeito fotoelétrico.
Antes que Millikan validasse experimentalmente
Figura 6 - O espectro eletromagnético,
essa lei em 1914. l instem foi indicado para membro
da Academia Prussiana de Ciências por Planck e mostrando o comprimento de onda, a frequência e a
outros. A atitude negativa inicial perante a hipótese" energia por fóton em escala logarítmica.
do fóton é revelada por eles em sua declaração,
57
Física Moderna – Capítulo 4 - Radiação de corpo negro e Efeito Fotoelétrico
Prof. Dr. Cláudio S. Sartori
58
Devido à grande massa de um átomo, ou
Figura 8 – Efeito fotoelétrico. Montagem
sólido, comparada com a do elétron, o sistema pode experimental.
absorver uma grande quantidade de momento sem
adquirir uma quantidade significativa de energia.
Nossa equação para a energia fotoelétrica permanece
válida, e o efeito é possível apenas porque existe uma
partícula pesada que recua além do elétron ejetado. O
efeito fotoelétrico é uma maneira importante pela qual
fótons, com energias que vão até às dos raios X
(inclusive), são absorvidos pela matéria. A energias
mais altas, outros processos de absorção de fótons, que
logo discutiremos, tornam-se mais importantes.
Finalmente, devemos enfatizar aqui que no
modelo de Einstein um fóton de frequência v tem
exatamente a energia hv, e não múltiplos inteiros de
hv. Evidentemente, pode haver n fótons de frequência
v de modo que a energia nessa frequência seja nhv. Ao
tratar a radiação de uma cavidade de corpo negro com
o modelo de Einstein, lidamos com um ''gás de
fótons", pois a energia radiante está localizada no
espaço em pacotes em vez de estar espalhada em
ondas estacionárias. Anos depois de Planck ter
deduzido sua fórmula para a radiação de cavidade,
Bose e Einstein obtiveram a mesma fórmula baseados
em um gás de fótons.
Figura 7 – Efeito fotoelétrico.
Figura 9 – Aplicação
fotoelétrico: Fotomultiplicadoras.
do
Efeito
58
Física Moderna – Capítulo 4 - Radiação de corpo negro e Efeito Fotoelétrico
Prof. Dr. Cláudio S. Sartori
Tabelaelementos.
59
Função
trabalho
para
  1090nm
diversos
Elemento
Função trabalho (eV)
Alumínio
4.3
Carbono
5.0
Cobre
4.7
Ouro
5.1
Níquel
5.1
Silício
4.8
Prata
2.7
Sódio
2.7
Este comprimento de onda está na região
do infravermelho do EE. A energia mínima de 1.14
eV corresponde ao comprimento de onda máximo
que produz fotocondutividade no silício, portanto
todos os fótons da luz visível também produzirão
fotocondutividade pois possuem comprimento de
onda menores do que esse limite e energias mais
elevadas do que aquela.
 Exemplo 5 – Uma experiência do
efeito fotolétrico. Realizando uma experiência do
efeito fotoelétrico com a luz de determinado
comprimento de onda, você verifica que é necessário
uma diferença de potencial invertida de 1.25 V para
anular a corrente. Determine:
(a) a energia cinética máxima;
(b) a velocidade máxima dos fotoelétrons
emitidos.

Solução:
 e V0  Kmax  1.6 1019 1.25
Kmax  1.25eV
1
2
(b) K max  me  vmax  K max  e  V0
2
2 K max
2  2 1019
vmax 
 vmax 
me
9.111031
m
vmax  6.63 105
s
(a) Kmax
Essa velocidade equivale a 1/500 da
velocidade
da luz; logo, podemos utilizar a equação
 Exemplo 4 - Um filme de silício torna-se
não
relativística
para a energia cinética.
um bom condutor elétrico quando iluminado com
fótons de energia de 1.14 eV ou superior (Este
 Exemplo 6 – Experiência para determinar
comportamento é denominado de fotocondutividade.)
e e h. Para um certo material do catodo na
Qual é o comprimento de onda correspondente?
experiência do efeito fotoelétrico, verifica-se um
potencial de corte de 1.0 V para uma luz de
 Solução:
comprimento de onda de 600 nm, 2.0 V para 400 nm
hc
hc
e 3.0 V para 300 nm. Determine a função trabalho
E


E
para esse material e a constante de Planck.
6.6262 1034  3 108
E
1.98786 1025
  m 
EJ 

Como
1eV  1.6 1019 J  1J  6.25 1018 eV
1.98786 1025
  m 
EJ 
1241.875
  nm  
E  eV 
1241.875
  nm  
1.14
 Solução:
Como e V0  h 
f 
59
Física Moderna – Capítulo 4 - Radiação de corpo negro e Efeito Fotoelétrico
Prof. Dr. Cláudio S. Sartori
h

V0   f 
e
e
são percebidos, fazendo com que a energia irradiada
pareça um fluxo contínuo.
A partir dessa forma,vemos que a inclinação
 Exemplo 8 – Luz do Sol. A
da reta é igual a h/e e a interseção com o eixo vertical superfície do Sol possui uma temperatura
(correspondente a f = 0) ocorre no ponto (0,-/e). As aproximadamente igual a 5800 K. Com boa
frequências obtidas pela relação c    f são:
aproximação, podemos considerá-la um corpo negro.
(a) Qual é o comprimento de onda max que
f (Hz)

fornece a intensidade do pico?
(nm)
(b) Qual é a potência total irradiada por
600
c
3 108
unidade de área?
15
f
60

f
600 109
 f  0.50 10
 Solução:
Usando a Lei do deslocamento de Wien:
0.75 1015
400
1.0 1015
300
Do gráfico:

m 

 1 (interseção vertical).
e
  1eV    1.6 1019 J
2.9 103  m  K 
2.9  mm  K 
 m 
T
T
3
2.9 10  m  K 
m 
 m  500nm
5800
Inclinação:
V0 3.0  (1.0)
V
J .s

 0  4.0 1019
15
f
110  0
f
C
h V0
h

  4.0 1019
e f
e
19
h  4.0 10  e  h  4.0 1019 1.6 1019
h  6.4 1034 J  s
(b) Usando a Lei de Stefan-Boltzman:
1
P    A  e T 4
P
I    T 4
A
I  5.67 108  58004
W
I  6.42 107 2
m
MW
I  64.2 2
m
 Exemplo 7 – Fótons de uma rádio FM.
Uma estação de rádio transmite ondas com frequência
89.3 MHz com potência total igual a 43.0 kW.
(a) Qual é o módulo do momento linear de
cada fóton?
Esse valor enorme indica a intensidade na
(b) Quantos fótons ela emite por segundo?
superfície do Sol. Quando a potência irradiada atinge
a Terra, a intensidade cai para 1400W/m2 porque ela
 Solução:
se espalha para fora do Sol e atinge a área muito
(a) Energia de cada fóton:
grande de uma superfície esférica cujo raio é a
34
6
distância entre a Terra e o Sol.
E  h  f  E  6.6262 10  89.3 10
E  5.92 1026 J
Cada fóton tem um momento linear dado por:
E
5.92 1026
 p
c
3 108
p  1.97 1034 kg  m s
p
(b) A estação emite 43.103J a cada segundo.
A taxa de emissão de fótons é:
nf 
43 103 J s
5.92 1026 J fóton
n f  7.26 1029 fótons s
 Exemplo 9 – Calcule a intensidade da luz
emitida da superfície do Sol no intervalo entre 600nm
e 605.5 nm. Lei da radiação de Planck:
I   
2  h  c 2
h c

5   k T
 e
 1


 Solução:
Como resultado exato:
2
I   I   d  
1
2  h  c 2
d

hc

600.0 5   k T
 e
 1


600.5
Como é muito grande o número de fótons que
Essa integral não pode ser calculada com
deixam a emissora de rádio a cada segundo, os saltos base em funções familiares. , logo, aproximamos a
de energia desses pequenos pacotes individuais não área pelo produto da altura medida no comprimento
60
Física Moderna – Capítulo 4 - Radiação de corpo negro e Efeito Fotoelétrico
Prof. Dr. Cláudio S. Sartori
de onda médio  = 602.5 nm vezes a largura do
intervalo ( = 5.0 nm). Inicialmente, calculamos:
34
c  3 108

6.62 10 J .s  3 10
hc

1  k  T 6.025 107 m 1.38110
 5800 K
8 m
s
23 J
K
hc
 4.116
1  k  T
I   
61
I    6.025 10 m  
7
2
6.6210 310


7
 23 5800
  e 6.02510 1.38110
 1




34

8
W
I    6.025 10 m   7.8110
m3
7
0
1
 A 
x   e x  1


dx
5
Essa integral vc não acha nem a pau!!
Sugestão:
http://integrals.wolfram.com
2  6.62 1034   3 108 
 6.025 10 


2  h  c 2
 h c

 5   e  k T  1


7 5
Solução:
m
s
13
A intensidade no intervalo de 5 nm entre os
limites de 600.0 nm e 605.0 nm é, aproximadamente
igual a:
I       7.811013  5 109
W
m2
MW
I       0.39 2
m
I       3.9 105

 d  
d  
2
c
c
2
d
d   
c

c
c
d  d   2 d
2
c 


2
2  h  c
c
0  c 5  h    2 d
k T
    e  1
  

d  
2


2  h
3
d
c 2 0 khT
e 1
Use agora:

Ou seja, 0.6% da intensidade luminosa total
proveniente do Sol se encontra entre os limites de
600.0 nm e 605.0 nm
c
x3
1  2 
0 e x  1dx  240   
4
 Exemplo 10 – Mostre que, usando a Lei da
radiação de Planck:
2  h  c 2
I   
 h c

 5   e  k T  1




2
2  h  c
2 5  k 4 4
I

d


d




0
0 5  hc  15c2  h3 T
   e  k T  1


E que, a constante de Stefan-Boltzmann é
dada por:
2 5  k 4

15c 2  h3
  5.6705 108
W
m K4
2
Com:
Constante de Boltzmann:
k  1.3811023
J
K
Constante de Planck:
h  6.62 1034 J .s
Velocidade da luz no vácuo:
61
Física Moderna – Capítulo 4 - Radiação de corpo negro e Efeito Fotoelétrico
Prof. Dr. Cláudio S. Sartori
62
62

Documentos relacionados

Cacilda Moura Espectroscopia e Evolução Estelar Espectro

Cacilda Moura Espectroscopia e Evolução Estelar Espectro Existem duas leis que descrevem a radiação emitida por um corpo negro: Qual a relação entre a quantidade de energia emitida e a temperatura ? Lei de Stefan - (lei empírica) Jožef Stefan (1835-1893)

Leia mais